Board study Flashcards
A 2-week-old boy is brought to the clinic for persistent vomiting. The neonate is exclusively fed a standard cow milk–based formula. He takes 4 oz every 2 hours and vomits after every feeding, despite burping. The emesis is not bloody, bilious, or projectile. He has 4 yellow, seedy, soft stools per day without blood. He is otherwise healthy. The neonate was born at term via an uncomplicated delivery and had an unremarkable newborn nursery course. His weight today is greater than his birth weight. His vital signs and physical examination findings are unremarkable.
Of the following, the MOST likely cause of this neonate’s symptoms is
A. gastroesophageal reflux disease B. milk protein allergy C. overfeeding D. pyloric stenosis
C. overfeeding
A 12-year-old, 40-kg boy in the pediatric intensive care unit with acute respiratory distress syndrome is currently intubated and mechanically ventilated, and has persistent hypoxemia. His last arterial blood gas analysis showed a pH of 7.22, pCO2 of 65 mm Hg, and a pO2 of 58 mm Hg. After adjusting the ventilator, his arterial blood gas analysis shows a pH of 7.25, pCO2 of 63 mm Hg, and pO2 of 80 mm Hg.
Of the following, the MOST likely change made to the mechanical ventilator settings was a(n)
A. decrease in positive end-expiratory pressure from 8 to 7 cm H2O B. decrease in tidal volume from 300 to 240 mL C. increase in inspiratory time from 0.7 to 0.9 seconds D. increase in respiratory rate from 22 to 24 breaths/minute
C. increase in inspiratory time from 0.7 to 0.9 seconds
The child in the vignette has acute respiratory distress syndrome (ARDS) and persistent hypoxemia. The change made to the mechanical ventilator settings resulted in an improvement in oxygenation (pO2) with negligible change in ventilation (pCO2). Oxygenation is a function of mean airway pressure. The main drivers of mean airway pressure are peak inspiratory pressure (PIP), positive end-expiratory pressure (PEEP), and inspiratory time fraction (i-time) (Item C2A). Thus, the most likely ventilator change that resulted in a higher pO2 was an increase in the inspiratory time fraction. Decreasing the PEEP would have resulted in the opposite effect and likely a lower pO2. A decrease in the tidal volume would not have increased the pO2 and would have caused an increase in pCO2. An increase in the respiratory rate would have little effect on oxygenation and would have decreased the pCO2.
An 18-month-old boy is brought to the emergency department by his parents after they found him in the garage coughing and sputtering next to an open bottle of lamp oil. Prior to arrival his parents removed the clothing that was soiled with lamp oil. The boy is awake, alert, and interactive. His vital signs include a heart rate of 125 beats/min, respiratory rate of 35 breaths/min, and blood pressure of 85/60 mm Hg. He has audible wheezing but no stridor and is in mild distress. The boy’s oral mucosa is pink and moist with no lesions. His skin is warm and well perfused, and he does not have a rash. The remainder of the boy’s physical examination findings are normal. A chest radiograph is obtained and shown in Item Q81.
Of the following, the BEST intervention for this boy is the administration of
A. activated charcoal B. corticosteroids C. ipecac syrup D. oxygen
The boy in the vignette has ingested and aspirated lamp oil, which contains hydrocarbons. Given that the boy has respiratory distress and bilateral infiltrative disease on his chest radiograph, oxygen should be administered immediately, and he should be admitted to the hospital for further observation and management.
PREP Pearls
Aspiration of hydrocarbon-containing substances is related to the hydrocarbon’s low viscosity and low surface tension.
Asymptomatic children seen after hydrocarbon exposure should be observed for at least 6 hours with serial examinations; if they remain asymptomatic they can be safely discharged to home.
Children with any respiratory symptoms after hydrocarbon exposure should receive oxygen and a trial of β<span>2</span>-agonists if there are signs of bronchospasm. Chest radiography should be performed.
Neither prophylactic antibiotics nor systemic corticosteroids should be administered routinely in pediatric hydrocarbon aspiration.
A term neonate had Apgar scores of 8 and 9 at 1 and 5 minutes, respectively, after birth. After delivery, he was active and crying, with room air oxygen saturations ranging from 93% to 97%. Approximately 10 minutes after birth, when he was quiet and calm, the neonate became cyanotic, his oxygenation saturation level decreased to 65% to 70%, and he developed significant respiratory distress with subcostal retractions and nasal flaring. He was treated with bag/mask positive-pressure ventilation using inspired fractionated oxygen of 100% without significant improvement in clinical status. He was, therefore, intubated, and positive-pressure ventilation was continued. The neonate’s clinical status improved almost immediately after intubation. His oxygen saturation increased to 100%; oxygen supplementation was quickly weaned and discontinued. He was placed on continuous positive airway pressure of 5 cm H2O. He is now pink, well perfused, and breathing comfortably in room air, with oxygen saturations of 95% to 98%.
Of the following, the BEST next step in this neonate’s management is to
A. obtain an arterial blood gas B. obtain echocardiography C. pass a nasogastric tube through both nares D. place an umbilical venous catheter
c
The neonate in the vignette most likely has bilateral choanal atresia. After delivery, he had no respiratory distress and had normal oxygen saturation levels. He quickly developed oxygen desaturation and respiratory distress when he became quiet and calm. The best next step in his management would be to pass a nasogastric tube through botThe neonate’s clinical status markedly improved once he was provided a stable airway through an endotracheal tube, which is suggestive of nasal obstruction rather than a cardiac lesion; therefore, echocardiography is not immediately indicated. An arterial blood gas and an umbilical venous catheter may be needed as part of ongoing care but would not be prioritized over the attempt to pass an nasogastric tube.
h nares. In the case of bilateral choanal atresia, the physician would be unable to pass the tube through either naris.
Bilateral choanal atresia in a neonate is a medical emergency because neonates are obligate nose breathers.
Passage of a nasogastric tube through both nares should be attempted in a neonate with suspected choanal atresia.
A neonate with choanal atresia or stenosis should undergo evaluation for associated anomalies, such as those seen in CHARGE syndrome
A 3-year-old boy is evaluated for a 1-month history of intermittent fevers up to 40°C, night sweats, and 2.2-kg weight loss. He has no known sick contacts and has not traveled recently. The boy is afebrile. He is irritable but consolable and appears well nourished and well developed. There are multiple subcentimeter cervical lymph nodes palpable, more on the left side than the right, and a 4 × 3–cm, firm, immobile, nontender, left-sided, supraclavicular lymph node. His spleen is palpable 1 cm below the left costal margin. The remainder of his physical examination findings are unremarkable.
Of the following, the BEST next step in this boy’s management is referral to a(n)
A. emergency department B. gastroenterologist C. infectious disease specialist D. otolaryngologist
The boy in the vignette has cervical and supraclavicular lymphadenopathy. Cervical lymphadenopathy is very common and has many possible etiologies. Supraclavicular lymphadenopathy, however, is highly concerning for malignancy (eg, lymphoma) and requires further investigation with a lymph node biopsy. Of the response choices, an otolaryngologist is best suited to perform a biopsy. Interventional radiologists and pediatric surgeons also perform lymph node biopsies, but these specialties are not among the response choices.
Supraclavicular lymphadenopathy is usually pathologic and requires further investigation with a lymph node biopsy.
Persistent or worsening lymphadenopathy requires further investigation.
A 13-month-old girl is seen for a health supervision visit. She was diagnosed with perinatally acquired HIV infection at 2 weeks of age and is currently receiving combination antiretroviral therapy. She has no history of opportunistic infections. She received her routine childhood immunizations at ages 2, 4, and 6 months including Haemophilus influenzae type b (Hib) conjugate vaccine and pneumococcal conjugate vaccine (PCV13). Her physical examination findings are normal. Laboratory data are notable for a CD4+ T lymphocyte count of 750 cells/µL (reference range for age 1-5 years, ≥1,000 cells/µL), CD4+ percentage of 25% (reference range for age 1-5 years, ≥30%), and an HIV viral load of 4,900 copies/mL.
Of the following, in addition to diphtheria-tetanus-pertussis (DTaP), Hib, PCV13, and hepatitis A, the MOST appropriate vaccine(s) to administer today is
A. measles-mumps-rubella B. measles-mumps-rubella and varicella C. measles-mumps-rubella-varicella D. varicella
The girl in the vignette is living with HIV infection and has evidence of low-level immunosuppression based on the absence of opportunistic infections, a CD4+ T lymphocyte count greater than 500/µL, and CD4+ percentage greater than 22%. In this setting, both measles-mumps-rubella (MMR) and varicella vaccines are indicated. In addition, all inactivated vaccines, including diphtheria-tetanus-pertussis (DTaP), Haemophilus influenzae type b (Hib), pneumococcal conjugate vaccine 13 (PCV13), and hepatitis A vaccines, should be administered at this visit. Given the lack of safety data, children with HIV infection should not receive the quadrivalent measles-mumps-rubella-varicella (MMRV) vaccine.
Administration of live vaccines, such as measles-mumps-rubella (MMR) and varicella, is recommended for children and adolescents living with HIV infection who have low-level or no immunosuppression.
Children living with HIV with high-level immunosuppression (defined as a CD4+ T lymphocyte percentage of less than 15% in children aged 1 through 13 years, or a CD4+ T lymphocyte count of less than 200 cells/µL in adolescents aged 14 years and older) must not receive measles-mumps-rubella (MMR) or varicella vaccines.
Children with immunosuppression, including HIV infection, should receive killed vaccines.
A 2-year-old girl with a noncontributory medical history is brought to the emergency department for intermittent abdominal pain over the past 12 hours. The pain, though progressively worsening, waxes and wanes. She had 3 episodes of nonbloody, nonbilious vomiting today. Over the past 6 hours, the girl has had a significantly decreased activity level and intermittent irritability.
Her vital signs include a temperature of 37.9°C, heart rate of 130 beats/min, respiratory rate of 24 breaths/min, and blood pressure of 108/76 mm Hg. On physical examination, the girl appears tired and cries intermittently while drawing her legs in toward her chest. She has right lower quadrant tenderness without rebound or guarding. The remainder of her physical examination findings are unremarkable.
Abdominal ultrasonography findings are shown in Item Q5.
Of the following, the BEST next step in this girl’s management is a(n)
A. air enema procedure B. computed tomography of the abdomen C. emergency laparotomy D. upper gastrointestinal imaging with small bowel series
The girl in the vignette has signs and symptoms concerning for intussusception. Abdominal ultrasonography demonstrates the characteristic “target sign” (Item C5), also known as a “bull’s eye” or “coiled spring,” which represents layering of intestine within the intestine. Because the girl is hemodynamically stable, performing an air enema is the best next management step. An air enema can be both diagnostic and therapeutic. If the air enema fails to reduce the intussusception, an emergency laparotomy may be necessary. Surgical intervention is indicated as the initial treatment if free air is visualized on imaging studies or there is evidence of peritonitis on physical examination.
PREP Pearls
The classic triad of abdominal pain, currant-jelly stools, and a palpable abdominal mass is present in less than 15% of children with intussusception.
An air enema can be both diagnostic and therapeutic for intussusception.
Extreme lethargy or altered mental status may be the only presenting symptom of intussusception.
A 2-year-old girl was seen for a routine health supervision visit 2 weeks ago. At that time, her weight was inadvertently entered into the medical record in pounds but labeled as kilograms. The girl’s mother called the advice line last night because her daughter had a fever and was provided an acetaminophen dose based on the incorrect weight. The pediatrician noticed the error when reviewing the chart this morning and called the girl’s mother for follow-up. The girl received 1 dose of acetaminophen since the overnight phone call, is feeling better, and is not exhibiting negative consequences of the dosing error. The dose administered was well below a hepatotoxic level.
Of the following, this event is BEST categorized as a/an
A. adverse event B. medical error C. non-preventable adverse event D. sentinel event
b
A 3-month-old male infant born at term is brought to the emergency department via ambulance for a seizure. Initial evaluation shows an ionized calcium level of 3.3 mg/dL (0.8 mmol/L) (reference range, 4.5-5.3 mg/dL [1.1-1.3 mmol/L]). He is treated with intravenous calcium, which aborts the seizure. The infant’s parents report that he has been jittery and irritable for the past few days. He exclusively breastfed for the first 2 months after birth. Due to fussiness, his parents recently started making formula using a recipe found on the internet containing hemp seed hearts, coconut water, dates, and sea moss. The infant was recently diagnosed with laryngomalacia after an evaluation for noisy breathing. His physical examination findings are normal for age. His parents are of normal stature.
Laboratory evaluation drawn prior to treatment reveals the following:
Laboratory Test
Result
Total calcium
4.5 mg/dL (1.1 mmol/L) (reference range, 9-11 mg/dL [2.2-2.8 mmol/L])
Phosphorous
1.2 mg/dL (reference range, 2.7-4.5 mg/dL)
Magnesium
2.1 mg/dL (0.9 mmol/L) (1.6-2.6 mg/dL [0.7-1.1 mmol/L])
Parathyroid hormone
534 pg/mL (reference range, 10-65 pg/mL)
Alkaline phosphatase
1,021 U/L (reference range, 146-477 U/L)
25-hydroxyvitamin D
Pending
1,25-dihydroxyvitamin D
Pending
Of the following, the BEST next step in this infant’s management is oral administration of
A. calcium, cholecalciferol, and calcitriol B. calcium and phosphorus C. magnesium D. phosphorus and calcitriol
The infant in the vignette has hypocalcemia due to severe vitamin D deficiency. His noisy breathing and seizure are manifestations of hypocalcemia. After treating his symptomatic hypocalcemia with intravenous calcium, the infant should be treated with oral calcium and cholecalciferol (dietary vitamin D3). Calcitriol (1,25-dihydroxyvitamin D, the active form of vitamin D) should be added given the severity of his hypocalcemia. Calcitriol acts immediately to absorb calcium from the intestine while vitamin D stores are replenished by cholecalciferol. The infant should also be switched to a cow milk–based formula, given its higher vitamin D content, once he can safely drink from a bottle.
Breast milk does not provide adequate vitamin D; infants who are exclusively or predominantly breastfed must receive supplementation with this vitamin. Although commercial formulas are fortified with vitamin D, homemade formulas, including the one described in the vignette, do not contain additional vitamin D. The American Academy of Pediatrics recommends that breastfed infants receive at least 400 IU of supplemental vitamin D daily.
Treatment with phosphorus and calcitriol is indicated for hypophosphatemia due to defects in the fibroblast growth factor 23 (FGF23) pathway, usually due to mutations in the PHEX gene. Fibroblast growth factor 23 is the main hormone responsible for phosphorus wasting in the kidney
In mild vitamin D deficiency, relative hypocalcemia will result in a rise in parathyroid hormone (PTH) to maintain serum calcium in the normal range. Calcium is reabsorbed from the kidney, absorbed from the gut (via conversion of remaining 25-hydroxyvitamin D stores to 1,25-dihydroxyvitamin D), and released from the bones. As vitamin D deficiency becomes more severe and PTH rises higher, calcium is depleted and levels begin to fall. Increased PTH levels cause phosphate wasting in the kidney, resulting in low serum phosphorus levels. Parathyroid hormone has an indirect effect on osteoclasts, resulting in increased bone resorption. Alkaline phosphatase levels rise as bone turnover is increased. When 25-hydroxyvitamin D stores are extremely low, there is inadequate vitamin D available for conversion to 1,25-dihydroxyvitamin D, which results in low levels of this hormone. The infant in the vignette is expected to have low 25-hydroxyvitamin D and 1,25-dihydroxyvitamin D levels
PREP Pearls
The American Academy of Pediatrics recommends supplementation of 400 IU daily of vitamin D for exclusively breastfed infants.
Manifestations of severe vitamin D deficiency may include rickets (widening of the wrists, rachitic rosary, craniotabes) and hypocalcemia with secondary hyperparathyroidism.
Homemade infant formulas may contain or lack ingredients that may affect mineral homeostasis.
A 12-day-old male neonate in the neonatal intensive care unit develops apnea, bradycardia, and oxygen desaturation requiring an increase in continuous positive airway pressure ventilation (CPAP) from 6 to 8 cm H2O, and fraction of inspired oxygen (FiO2) from 0.23 to 0.40. His mean upper limb cuff blood pressure has ranged from 18 to 25 mm Hg for the past hour. The neonate was born at 26 weeks’ gestation with a birthweight of 940 g. Delivery was by cesarean section due to maternal preeclampsia with rupture of membranes at delivery. The neonate has been tolerating nasogastric feedings of 20 calorie/oz breast milk at 80 mL/kg/day and receiving parenteral nutrition through a central intravenous line.
Laboratory data are shown:
Laboratory Test
Result
Arterial blood gas base deficit
10
White blood cell count
22,000/µL (22.0 × 109/L)
Neutrophils
40%
Bands
25%
Lymphocytes
20%
Monocytes
11%
Eosinophils
4%
Hemoglobin
10.8 g/dL (108 g/L)
Hematocrit
30.5%
Platelet count
87 × 103/µL (87 × 109/L)
Blood cultures are pending.
Of the following, the BEST treatment for this neonate is intravenous
A. ampicillin and cefotaxime B. ampicillin and gentamicin C. meropenem and oxacillin D. vancomycin and gentamicin
The preterm neonate in the vignette has signs and symptoms of late-onset sepsis with clinical deterioration and an elevated white blood cell count with a left shift. The most likely causative organisms are coagulase-negative staphylococci and gram-negative bacteria. Of the response choices, the combination of vancomycin (effective against coagulase-negative staphylococci) and gentamicin (effective against gram-negative bacteria such as Escherichia coli and Klebsiella pneumoniae) is the best treatment pending blood culture results.
Ampicillin, gentamicin, cefotaxime, and meropenem are all effective against gram-negative bacteria, but they are not effective against coagulase-negative staphylococci. Oxacillin is also not effective against coagulase-negative staphylococci. Ampicillin and gentamicin is the empiric antibiotic combination of choice in early-onset sepsis and is effective against the common bacterial causes of early onset sepsis (eg, group B Streptococcus, E coli, and Listeria monocytogenes).
PREP Pearls
Risk factors for late-onset sepsis in preterm neonates include immature defense barriers, an immature immune system, invasive interventions, and comorbidities of prematurity.
The choice of empiric antibiotics for late-onset sepsis in a preterm neonate should be based on the most likely causative organisms, which include coagulase-negative staphylococci and gram-negative bacteria.
The mother of a 10-year-old girl with cystic fibrosis (CF), jointly managed by her pediatrician and a multidisciplinary CF center at a nearby children’s hospital, calls for advice about a positive airway culture. The girl was diagnosed with CF after a positive newborn screening result. She has been very healthy and has required no aggressive intervention for pulmonary infection. Her routine airway cultures, obtained in the CF center by deep throat/gag swab, have always grown normal flora or methicillin-sensitive Staphylococcus aureus. Her most recent culture was positive for Pseudomonas aeruginosa. The girl has no symptoms of illness. She is eating well and has had no weight loss. She is attending school regularly.
Of the following, the MOST appropriate management strategy in response to this girl’s laboratory finding is
A. hospitalization for intravenous antibiotic administration B. inhaled antibiotic treatment for eradication of the identified organism C. no intervention while she is asymptomatic D. repeat airway culture before implementing treatment
The most appropriate management strategy for the girl in the vignette, who is experiencing her first acquisition of Pseudomonas aeruginosa infection, is an inhaled anti-pseudomonal antibiotic. Multiple studies demonstrate that proactive treatment to eradicate Pseudomonas from the airway in children with cystic fibrosis (CF) helps preserve lung function. A single 28-day treatment course of inhaled tobramycin given twice daily, with follow-up culture to document resolution and re-treatment if the culture is still positive, is the standard of care in the United States. The addition of oral antibiotics for acute eradication has not been shown to be more effective than an inhaled antibiotic alone. However, concurrent oral azithromycin has been demonstrated to delay symptomatic pulmonary exacerbation of cystic fibrosis in children with first acquisition of Pseudomonas. There are no data to support hospitalization for treatment with intravenous antibiotics as more effective than treatment with an inhaled antibiotic alone for eradication of Pseudomonas in a child with asymptomatic first acquisition.
PREP Pearls
Chronic colonization with Pseudomonas aeruginosa and/or methicillin-resistant Staphylococcus aureus is associated with a decline in lung function in children and adolescents with cystic fibrosis.
The most appropriate treatment of first acquisition of Pseudomonas in an individual with cystic fibrosis is 28 days of inhaled antipseudomonal antibiotic, usually tobramycin.
Cystic fibrosis-related diabetes is a later-onset complication of progressive pancreatic fibrosis.
A 5-year-old girl is seen for a health supervision visit. She attends kindergarten and is doing very well. Her medical history is significant for recurrent joint subluxations of the hips, shoulders, knees, and elbows bilaterally, easy bruising, and poor wound healing often requiring placement of sutures to achieve closure. The family history is significant for similar findings in her father and grandfather. The girl’s father had a recurrent incisional hernia at an appendectomy site. There is no family history of arterial aneurysm or rupture. On physical examination, there are bruises on her lower extremities, widened atrophic scars with evidence of previous sutures (Item Q10), and skin and joint hypermobility. The remainder of her physical examination findings are normal.
Of the following, the girl’s MOST likely diagnosis is
A. Ehlers-Danlos syndrome B. homocystinuria C. Loeys-Dietz syndrome D. Marfan syndrome
The girl in the vignette has classical Ehlers-Danlos syndrome (cEDS). Ehlers-Danlos syndrome is a group of inherited connective tissue disorders. Cardinal features of EDS include joint hypermobility, skin hyperextensibility, and tissue fragility. The 2017 international classification describes 13 subtypes of EDS. The most common forms are classical, vascular, and hypermobile. There is wide phenotypic variability and genetic heterogeneity among the different EDS subtypes. The underlying genetic etiology is known for all subtypes of EDS except hypermobile EDS. Classical Ehlers-Danlos syndrome is inherited in an autosomal dominant manner. The clinical diagnosis of cEDS is made based on the presence of major and minor criteria.
Homocystinuria is characterized by the following features:
Central nervous system: developmental delay, intellectual disability
Eyes: ectopia lentis, severe myopia
Vascular system: thromboembolic episodes
Skeletal: tall stature, long limbs, scoliosis, pectus excavatum
Clinical features of Loeys-Dietz syndrome are described below:
Craniofacial: hypertelorism, bifid uvula/cleft palate, strabismus, craniosynostosis
Cutaneous: velvety, translucent skin, easy bruising, dystrophic scars
Skeletal: pectus excavatum/carinatum, scoliosis, joint hypermobility, arachnodactyly, cervical spine malformation, clubfeet
Vascular: cerebral, thoracic, and abdominal arterial aneurysms and/or dissections
A 4-year-old girl is evaluated in the emergency department for 6 days of bloody diarrhea. Her mother reports that her daughter appears tired and pale. The girl drank only 4 oz of fluids today and has not urinated for more than 24 hours. The girl’s 2 older sisters also have diarrhea, but their symptoms are improving. The family recently vacationed at a cabin on a lake in a rural area. They did not drink any unfiltered lake water but drank the tap water.
The patient has a heart rate of 145 beats/min, a respiratory rate of 20 breaths/min, and blood pressure of 85/55 mm Hg. She is ill-appearing, and her capillary refill time is 4 seconds. The remainder of her physical examination findings are unremarkable.
Of the following, the MOST likely cause of this girl’s illness is
A. Escherichia coli B. Giardia duodenalis C. norovirus D. Yersinia enterocolitica
The girl in the vignette has bloody diarrhea that most likely resulted from drinking contaminated well water, a common source of water in rural areas. The most likely cause of her illness is infection with Escherichia coli (possibly hemolytic uremic syndrome). E coli, including E coli 0157:H7 and other forms of Shiga toxin–producing E coli, are common well-water contaminants. Drinking water contaminated with Giardia duodenalis, norovirus, and Yersinia enterocolitica can also cause diarrhea but is less likely to produce bloody diarrhea.
A 14-year-old adolescent girl is evaluated in the emergency department for fever of 3 days’ duration, sore throat, dysphagia, and a slightly muffled voice. She is otherwise healthy, fully immunized, and does not have a history of recurrent throat infections. Her family history is unremarkable. On physical examination, her temperature is 39°C, heart rate is 100 beats/min, respiratory rate is 18 breaths/min, and oxygen saturation is 99% in room air. She is in mild discomfort due to her throat pain but is speaking comfortably in complete sentences. Her left tonsil is 3+ enlarged and erythematous, her right tonsil is 1+, and her uvula is deviated to the right (Item Q14). She has enlargement and tenderness of her left cervical lymph nodes. The remainder of her physical examination findings are unremarkable.
Of the following, the MOST appropriate next management step for this adolescent is
A. drainage of the abscess
B. intravenous antibiotics
C. observation only
D. tonsillectomy
The adolescent in the vignette has a peritonsillar abscess (PTA); drainage of the abscess is the most appropriate first step in treatment. Intravenous antibiotics may be administered as the first step in treatment for individuals with certain complications resulting from PTAs (eg, septic thrombophlebitis). Observation alone is not appropriate management and may result in complications. Tonsillectomy for an acute infected PTA may be performed in certain individuals, such as those who would require general anesthesia for incision and drainage and those with a history of recurrent tonsil infections, but would not be the appropriate first-line treatment for this otherwise healthy adolescent.
A 13-year-old adolescent girl is seen in the office for evaluation of lower abdominal pain and hematochezia of 3 months’ duration. Her symptoms have progressed from mild lower abdominal pain and loose stools to severe, crampy lower abdominal pain with bloody, liquid stools 6 to 8 times per day. She has urgency and tenesmus, and passes nocturnal stools. Her family has noted pallor over the last few weeks, and she reports dizziness. The girl is otherwise healthy and does not take any medication. Her mother has rheumatoid arthritis and thyroid disease.
The girl is pale and quiet. She is afebrile, with a heart rate of 150 beats/min, blood pressure of 110/65 mm Hg, weight of 38 kg (13th percentile for age), height of 146 cm (4th percentile for age), and body mass index of 18 kg/m2 (30th percentile for age). She has conjunctival pallor, dry mucous membranes, and a soft systolic murmur. Her abdomen is soft, tender to palpation diffusely without rebound or guarding, and grossly bloody stool is noted on the rectal examination.
Laboratory evaluation in the office demonstrates a hemoglobin level of 7.8 g/dL (78 g/L) and positive stool occult blood test.
Of the following, the BEST next step in the management of this adolescent is
A. immediate referral to the local emergency department for evaluation and care B. oral iron supplementation with repeat hemoglobin test in 1 week C. referral to pediatric gastroenterology for appointment within 1 week D. stool infection studies with follow-up office appointment in 24 hours
The adolescent in the vignette has significant anemia, evidence of ongoing lower gastrointestinal bleeding, and vital signs that are concerning for hypovolemia (tachycardia). She needs to be urgently evaluated and stabilized in the emergency department.
The initial step in the evaluation of children with gastrointestinal bleeding (either through vomitus or stool) is the assessment of hemodynamic stability based on vital signs (tachycardia, hypotension, orthostatic changes), capillary refill, and mental status. If hemodynamic instability is noted, appropriate intravenous (IV) access and IV fluid will be necessary, with consideration of transfusions of packed red blood cells, platelets, and/or coagulation factors as clinically appropriate. Laboratory data, including hemoglobin level, platelet count, coagulation studies, liver function tests, albumin level, and type and cross-match (in preparation for potential blood transfusions), should be obtained. The presence of blood should be confirmed with a stool occult blood test, as stool or vomitus can appear red with the ingestion of certain foods, dyes, and medications. Urgent consultation with a pediatric gastroenterologist is indicated because children with refractory gastrointestinal bleeding and ongoing hemodynamic instability despite the administration of IV fluids and blood products may require emergent therapeutic endoscopy.
PREP Pearls
Initial assessment of a child with gastrointestinal bleeding includes evaluation of hemodynamic stability.
Children with gastrointestinal bleeding and hemodynamic instability need urgent intravenous access and intravenous fluid administration, and may require transfusion of blood products.
Upper gastrointestinal bleeding may manifest as hematemesis, melena, or hematochezia. Lower gastrointestinal bleeding may present as melena or hematochezia.
A 4-month-old, full-term, developmentally normal infant is evaluated in the outpatient office for paroxysmal episodes for the past 2 weeks. During the episodes, his body crunches up repeatedly, “as though he is doing baby sit-ups.” Initially, it occurred once or twice per day, usually in the evening, but it now occurs in clusters, at which point he becomes difficult to console. His mother suspects that he is uncomfortable and might have gastroesophageal reflux. She is concerned because the episodes are becoming more frequent.
The infant’s vital signs are normal. His physical examination findings, including neurological, are normal. During the visit, the infant had a typical episode in which he stiffened, his arms extended out suddenly and he bent forward. This occurred in a short cluster during which he appeared uncomfortable and cried.
Of the following, the BEST next step in this infant’s management is
A. referral to a gastroenterologist for outpatient evaluation B. referral to a neurologist for outpatient evaluation C. reflux precautions and close follow-up in the pediatrician’s office D. urgent neurological evaluation through the emergency department
The infant in the vignette’s episodes are suggestive of infantile spasms (IS), a condition requiring prompt diagnosis and treatment to optimize developmental outcome. Infantile spasms, the most common epilepsy syndrome in infancy, is clinically characterized by a triad of:
Epileptic spasms
Electroencephalogram background of hypsarrhythmia
Accompanying developmental plateau and regression
PREP Pearls
Infantile spasms, the most common epilepsy syndrome in infancy, are clinically characterized by a triad of: 1) epileptic spasms, 2) electroencephalogram background of hypsarrhythmia, and 3) accompanying developmental plateau and regression.
A high index of suspicion and urgent evaluation with expedited electroencephalography are necessary to confirm the diagnosis of infantile spasms and quickly initiate treatment.
Targeted evaluation for underlying causes of infantile spasms, focusing on treatable conditions, should occur concomitantly with treatment initiation and is tailored toward the individual patient. Long-term developmental outcome is impacted by time to treatment and underlying etiology.
A 5-year-old, previously healthy boy is seen in the clinic for evaluation of testicular pain and fever that started this morning. He had a cough and runny nose 2 weeks ago, which have resolved. He has no other symptoms and no history of trauma. On physical examination, the boy has a temperature of 38.5°C, heart rate of 117 beats/min, respiratory rate of 22 breaths/min, and oxygen saturation of 98% in room air. Both testicles are palpated in the scrotum and the cremasteric reflex is intact. The testicles appear swollen, with no abnormal coloring or masses palpated. Ultrasonography with Doppler shows normal blood flow to both testicles.
Of the following, this boy’s MOST likely diagnosis is
A. inguinal hernia B. postinfectious orchitis C. testicular torsion D. torsion of the appendix testis
The boy in the vignette has fever, painful testicular swelling, and a normal cremasteric reflex. Of the response choices, these signs and symptoms are most consistent with orchitis. In this age group, orchitis is most commonly seen after viral infections. An inguinal hernia can cause scrotal pain and swelling but would be palpated during the genitourinary examination. Testicular torsion, a surgical emergency, is an unlikely diagnosis for the boy in the vignette given his normal blood flow on Doppler ultrasonography and normal cremasteric reflex. In the case of testicular torsion, the normal cremasteric reflex (retraction of the testis in response to touch on the upper thigh) is usually absent. Torsion of the appendix testis is also unlikely given the normal findings on ultrasonography and absence of a “blue dot sign” on physical examination.
Orchitis (inflammation of the testis) and epididymitis (inflammation of the epididymis) have infectious or inflammatory etiologies. Symptoms and signs of orchitis and epididymitis include testicular swelling and tenderness, dysuria, urinary frequency and urgency, and often systemic symptoms (eg, fever). The etiology of orchitis and epididymitis varies by age. Children ages 2 to 13 years often have a postinfectious cause (most commonly Mycoplasma, enterovirus, or adenovirus). Vasculitis (eg, Henoch-Schönlein purpura) is another common cause of orchitis and epididymitis in this age group. Older children and adults are more likely to have an infectious etiology with organisms that cause urinary tract or sexually transmitted infections. A urinalysis may show the presence of nitrites and/or leukocyte esterase because the infection can start in the urine and ascend into the epididymis and testes.
PREP Pearls
All children and adolescents with testicular pain should have ultrasonography with Doppler performed to evaluate for testicular torsion.
Symptoms and signs of orchitis and epididymitis typically include testicular swelling and tenderness, dysuria, urinary frequency and urgency, and often systemic symptoms.
A 9-month-old girl is seen for follow-up 3 weeks after hospital admission for a febrile Escherichia coli urinary tract infection. She received intravenous ceftriaxone in the hospital and subsequently completed treatment with oral cefdinir. The girl is currently asymptomatic, feeding well, and having regular bowel movements. At 6 months of age she had a febrile urinary tract infection. She is alert and active, and has vital signs that are normal for age. Her physical examination findings, including those of the genitourinary examination, are unremarkable.
Renal ultrasonography performed in the hospital was normal. A voiding cystourethrogram showed bilateral grade III vesicoureteral reflux.
Of the following, the MOST appropriate next management step for this infant is
A. polyethylene glycol treatment, orally B. repeat voiding cystourethrogram in 3 months C. trimethoprim-sulfamethoxazole prophylaxis, orally D. ureteral reimplantation surgery
The infant in the vignette has recurrent febrile urinary tract infection (UTI) and a voiding cystourethrogram (VCUG) that shows bilateral vesicoureteral reflux (VUR). The most appropriate next management step for this girl is trimethoprim-sulfamethoxazole prophylaxis to prevent recurrent UTI.
Vesicoureteral reflux is the retrograde passage of urine from the bladder to the upper urinary tract. Vesicoureteral reflux is divided into primary and secondary types. Primary VUR occurs due to a congenitally short segment of ureter within the bladder wall, which results in incomplete closure of the ureterovesical junction during bladder contraction. Secondary VUR occurs as a result of high bladder pressure associated with conditions such as posterior urethral valves, bladder-bowel dysfunction, and neurogenic bladder.
Voiding cystourethrogram is the test of choice to confirm the presence of VUR and grade its severity. Indications to perform a VCUG in neonates with a history of antenatal hydronephrosis include presence of moderate to severe hydronephrosis or ureteral dilation on the postnatal ultrasonography. A VCUG is also indicated in a child with a first febrile UTI with abnormal renal ultrasonography, or a child with recurrent febrile UTI.
PREP Pearls
Vesicoureteral reflux may be diagnosed during the evaluation of a febrile urinary tract infection, antenatally diagnosed hydronephrosis, or screening in children with family history.
Spontaneous resolution of vesicoureteral reflux is common when it is low grade, unilateral, and in those with asymptomatic antenatally diagnosed hydronephrosis.
Antibiotic prophylaxis is indicated for vesicoureteral reflux in non–toilet-trained children, those with low-grade vesicoureteral reflux with recurrent urinary tract infection, and those with high-grade vesicoureteral reflux.
A 12-year-old boy is seen in the office for evaluation of bilateral anterior knee pain. His pain began about 3 months ago during wrestling practice. There was no acute injury. He reports no swelling, locking, or instability. The pain is worse with kneeling, running, and jumping. There is tenderness over the inferior aspect of both patellae. He also has pain with resisted knee extension. The remainder of the boy’s physical examination findings are unremarkable.
Of the following, the MOST likely cause of this boy’s pain is
A. Osgood-Schlatter disease B. patellar tendinopathy C. prepatellar bursitis D. Sinding-Larsen-Johansson syndrome
The most likely diagnosis for the boy in the vignette is Sinding-Larsen-Johansson syndrome (SLJ) or inferior patellar pole apophysitis. An apophysis is a bony ossification center adjacent to a minor growth plate. Apophyses are found at sites where tendons attach to bone. Contraction of a muscle causes the tendon to pull on the apophysis. The physis, the growth area adjacent to the apophysis, is made of soft bone that has not yet calcified and is especially vulnerable to injury. With repetitive activity, traction on the apophysis can lead to pain. Overuse of the quadriceps muscles and direct pressure on the bottom of the patella (eg, kneeling) cause irritation of the apophysis, known as apophysitis.
PREP Pearls
Apophysitis occurs because the physis, the growth area adjacent to the apophysis, is made of soft bone and is especially vulnerable to injury.
Sinding-Larsen-Johansson syndrome is an apophysitis of the inferior patellar pole apophysis. Osgood-Schlatter disease is an apophysitis of the tibial tubercle.
Tendinopathy and bursitis, while frequently seen in adults, are uncommon causes of knee pain in children.
A 20-month-old, previously healthy girl is brought to the urgent care center with a 1-day history of left otalgia and increased fussiness. She has had rhinorrhea and cough for the past week, and developed a fever of 38.8°C 6 hours ago that resolved after 1 dose of acetaminophen. There has been no otorrhea. She is drinking, eating, stooling, and voiding well. She has no past history of otitis media. On physical examination, she is afebrile and has clear rhinorrhea in both nares and 2 tender, mobile, soft, 1-cm posterior cervical lymph nodes. The appearance of her left tympanic membrane is shown in Item Q21 (only ear effusion). The remainder of her physical examination findings are normal.
Of the following, the BEST next step in this girl’s management is
A. high-dose amoxicillin for 5 days B. high-dose amoxicillin for 7 days C. high-dose amoxicillin for 10 days D. observation for 48 to 72 hours
The girl in the vignette has acute otitis media (AOM), which is an acute bacterial infection of the middle ear with fluid, otherwise known as suppurative otitis media. According to the American Academy of Pediatrics Acute Otitis Media Clinical Practice Guidelines, the best treatment for children younger than 24 months of age without severe signs or symptoms is observation for 48 to 72 hours with analgesic administration as needed followed by an antibiotic prescription if symptoms worsen or persist over that time frame.
The diagnostic criteria for AOM include:
Recent onset of ear pain (<48 hours)
Bulging tympanic membrane
Reduced mobility of the tympanic membrane with pneumatic otoscopy or tympanometry
Severe disease is defined as one of the following:
Otalgia that is moderate or severe in nature lasting for at least 48 hours
Temperature of 39°C (102.2°F) or higher
PREP Pearls
The diagnostic criteria for acute otitis media include an acute onset of ear pain (<48 hours), a bulging tympanic membrane, and reduced mobility of the tympanic membrane with pneumatic otoscopy.
Severe acute otitis media is defined as otalgia that is moderate or severe in nature lasting for at least 48 hours, or a temperature of 39°C (102.2°F) or higher.
For children aged 6 months to less than 2 years with unilateral acute otitis media that is not severe, observation for 48 to 72 hours is appropriate, after which antibiotic treatment is indicated for persistent or worsening symptoms.
A newborn is examined 4 hours after birth in the normal newborn nursery. She was born after 39 weeks’ gestation in an uncomplicated vaginal delivery to a 22-year-old woman with no significant medical history. Apgar scores were 8 and 9 at 1 and 5 minutes, respectively. On physical examination, the neonate has a soft boggy swelling over the occipital area that crosses suture lines with bruising and scattered petechiae on the scalp. The remainder of the physical examination findings are normal.
Of the following, the BEST next step in the management of this condition is
A. head ultrasonography B. serial hemoglobin levels C. serial physical examinations D. serum bilirubin levels
c
PREP Pearls
Caput succedaneum is a common newborn scalp swelling found immediately after birth.
Caput succedaneum typically crosses suture lines and resolves spontaneously within 48 to 72 hours.
Due to the risk of severe blood loss associated with subgaleal hemorrhage, which results from tearing of the bridging veins connecting the scalp and the intradural sinuses, it is important to differentiate this condition from caput succedaneum.
A 3-year-old girl is admitted to the hospital for evaluation and treatment of acute-onset systemic hypertension with an arterial blood pressure of 130/88 mm Hg. She is otherwise healthy. A parenteral antihypertensive medication is prescribed for administration every 6 hours in order to achieve optimum efficacy once the medication serum concentration is at steady state. The elimination half-life of the medication is 6 hours and the medication follows first-order kinetics. After the second dose, the girl remains hypertensive with a blood pressure of 120/65 mm Hg but is otherwise asymptomatic.
Of the following, the MOST likely time-frame, after administration of the initial dose, for this medication to achieve optimum efficacy is
A. 12 hours B. 18 hours C. 24 hours D. 30 hours
The medication administered to the girl in the vignette follows first-order kinetics and has a 6-hour elimination half-life. When given in regular intervals, this medication will achieve steady state concentration around 30 hours after the initial dose (6-hour elimination half-life × 5 half-lives). Elimination half-life is defined as the time it takes for the plasma concentration of the drug or the total amount of drug in the body to be reduced by 50%. It is only applicable to drugs that exhibit first-order kinetics, in which a constant fraction of drug is eliminated per unit time. The steady state of a medication concentration is achieved after the fifth half-life. It takes 5 elimination half-lives for ~97% of the bioavailable medication dose to be eliminated from the body. Thus, in this vignette, it is not surprising that after the second dose this girl has not achieved optimal blood pressure control.
A 2-month-old infant is brought to the emergency department in January for trouble breathing. He has had progressively worsening rhinorrhea and cough for the past 2 days. Over the past 8 hours, he has not been tolerating his breast milk feedings due to congestion and rapid breathing. The infant was born at 31 weeks’ gestation and has otherwise been healthy.
On physical examination, he has a temperature of 38.0°C, heart rate of 149 beats/min, respiratory rate of 46 breaths/min, and oxygen saturation of 89% in room air. He appears to be in mild respiratory distress. Copious clear rhinorrhea and subcostal retractions are present. End expiratory wheezing is heard in both lung fields. The remainder of his physical examination findings are normal.
Of the following, the MOST likely pathogen causing this infant’s illness is
A. coronavirus B. influenza virus C. parainfluenza virus D. respiratory syncytial virus
The history and physical examination findings of the infant in the vignette, with an antecedent upper respiratory tract infection, low-grade fever, and respiratory distress, are consistent with the diagnosis of acute bronchiolitis. Bronchiolitis is a common inflammatory illness of the lower respiratory tract in infants and young children. Although many community-acquired respiratory viruses can cause bronchiolitis, respiratory syncytial virus (RSV) is the most common cause in infants followed by human rhinovirus, human metapneumovirus, and parainfluenza virus. Other viral pathogens such as influenza and coronavirus rarely cause bronchiolitis. In addition, influenza is associated with high-grade fever.
A 16-year-old adolescent girl is brought to the emergency department for evaluation of frequent voiding for 2 months. She has 10 to 12 large-volume voids in a day. She has no burning with urination or urinary incontinence. She typically drinks 5 to 6 L of water per day. The girl’s heart rate is 82 beats/min, respiratory rate is 16 breaths/min, and blood pressure is 110/70 mm of Hg. Her weight is at the 10th percentile and height is at the 25th percentile for age. She has no neurologic deficits, and the rest of the examination findings are unremarkable.
Laboratory data are shown:
Blood
Result
Sodium
133 mEq/L (133 mmol/L)
Potassium
4.1 mEq/L (4.1 mmol/L)
Chloride
98 mEq/L (98 mmol/L)
Bicarbonate
24 mEq/L (24 mmol/L)
Blood urea nitrogen
7 mg/dL (2.5 mmol/L)
Creatinine
0.6 mg/dL (53 μmol/L)
Glucose
72 mg/dL (4 mmol/L)
Serum osmolality
270 mOsm/kg
(reference range, 275-305 mOsm/kg)
Urine
Result
Specific gravity
1.005
Leukocyte esterase
Negative
Nitrite
Negative
Blood
Negative
Protein
Negative
Glucose
Negative
Urine osmolality
150 mOsm/kg
(reference range, 300-900 mOsm/kg)
Of the following, the MOST likely diagnosis for this adolescent girl is
A. diabetes insipidus B. diabetes mellitus C. primary polydipsia D. syndrome of inappropriate antidiuretic hormone
The girl in the vignette has polyuria, polydipsia, low serum sodium, low serum osmolality, and dilute urine, favoring a diagnosis of primary polydipsia.
Polyuria is defined as urine volume production of more than 2 L/m2 or 40-50 mL/kg in 24 hours. Primary polydipsia is characterized by the consumption of an excessive amount of fluids, leading to polyuria and dilute urine. It is commonly seen in individuals with psychiatric conditions (eg, schizophrenia, depression, and bipolar disorders) and is often referred to as psychogenic polydipsia. Primary polydipsia can be seen in healthy individuals who drink large quantities of water out of habit; this condition is referred to as habitual polydipsia or compulsory water drinking.
The differential diagnosis of primary polydipsia includes diabetes insipidus, which also presents with polyuria, polydipsia, and dilute urine. The child in the vignette has hyponatremia (hypo-osmolality), whereas the serum sodium level is high or normal in diabetes insipidus. A water deprivation test can be performed to differentiate primary polydipsia from diabetes insipidus when the diagnosis is not clear. In primary polydipsia, polyuria decreases, and the urine osmolality increases after water deprivation. However, in diabetes insipidus, polyuria persists, and the urine osmolality is low after water deprivation.
PREP Pearls
Primary polydipsia is characterized by consumption of an excessive amount of fluids, leading to polyuria, low serum sodium level, low serum osmolality, and dilute urine.
Primary polydipsia can be asymptomatic or associated with symptoms of hyponatremia (eg, nausea, vomiting, lethargy, confusion, ataxia, and seizures).
The treatment for primary polydipsia is restriction of water intake.
A 10-month-old female infant is brought for a health supervision visit. She was last seen 4 months ago. She has been well since that visit, with no significant illness. She has been feeding well, taking 4 to 5 ounces of formula 4 to 5 times/day plus pureed foods and yogurt. At birth, her weight was at the 30th to 40th percentile; at her previous health supervision visit, her weight was at the 15th to 20th percentile. She says “mama” and “dada,” and is able to pull up to stand but does not crawl. She is followed by cardiology for a heart murmur. The girl’s temperature is 36.5°C, heart rate is 100 beats/min, respiratory rate is 28 breaths/min, blood pressure is 96/45 mm Hg, and oxygen saturation is 100% in room air. Her weight is 7.3 kg (8th percentile) and length is 65 cm (<3rd percentile). She appears pale and is in no apparent distress. There is a 3/6 systolic ejection murmur, lungs are clear to auscultation bilaterally, and her abdomen is soft and nontender with no hepatosplenomegaly. There are no focal neurologic deficits. The remainder of her physical examination findings are normal.
Laboratory data are shown:
Laboratory Test
Result
White blood cell count
7,400/µL (7.4 × 109/L)
Hemoglobin
3.2 g/dL (32 g/L)
Platelet count
314 × 103/µL (314 × 109/L)
Mean corpuscular volume
115 fL
Neutrophils
22%
Lymphocytes
71%
Monocytes
7%
Reticulocyte
0.8%
Absolute reticulocyte count
6 × 103/µL
Hemoglobin A
81.6%
Hemoglobin A2
2.4%
Hemoglobin F
16%
Adenosine deaminase
1.98 (reference range, 0.33-0.95 U/g of hemoglobin)
Of the following, the MOST appropriate treatment for this girl’s condition is
A. erythropoietin administration B. long-term transfusion therapy C. no intervention with continued observation D. single transfusion with continued observation
The girl in the vignette has a history and laboratory findings that are consistent with a pure red blood cell aplasia, most likely Diamond-Blackfan anemia (DBA). This diagnosis is supported by severe macrocytic anemia without reticulocytosis, elevated fetal hemoglobin level, and elevated erythrocyte adenosine deaminase (eADA) level. Her cardiac murmur may be evidence of a congenital heart defect (CHD); there is an increased prevalence of congenital anomalies (eg, cardiac, genitourinary, thumb) in children with DBA. Genetic testing may reveal a mutation in a ribosomal protein, however, not all mutations have been identified.
The treatment for DBA is long-term blood transfusion therapy, which is required to support the child’s growth. Leucodepleted red blood cells should be administered to avoid febrile reactions, cytomegalovirus transmission, and human leukocyte antigen (HLA) alloimmunization. It is important to obtain a blood sample for hemoglobin electrophoresis and eADA testing before administering a blood transfusion, as those tests must be performed on the child’s native blood cells and would otherwise not be accurate until 3 months after a blood transfusion.
PREP Pearls
Diamond-Blackfan anemia is a rare form of pure red blood cell aplasia with severe macrocytic anemia without reticulocytosis, elevated fetal hemoglobin level, and elevated erythrocyte adenosine deaminase level.
There is an increased prevalence of congenital anomalies (eg, cardiac, genitourinary, thumb) in children with Diamond-Blackfan anemia.
Transient erythroblastopenia of childhood is a form of pure red blood cell aplasia that most often occurs in very young children (typically >1 year of age) after a viral infection. The condition spontaneously resolves within a few weeks.
A 13-year-old adolescent boy is being seen for follow-up after an urgent care visit several days ago for 1 week of fatigue preceded by 2 days of fever and sore throat. Results of the laboratory tests obtained at that visit are shown:
Laboratory Test
Result
Complete blood cell count with differential
Normal with mildly elevated atypical lymphocytes
Rapid streptococcal antigen
Negative
Heterophile antibody spot test for infectious mononucleosis
Positive
Epstein-Barr virus capsule antigen IgG
Positive
Epstein-Barr virus capsule antigen IgM
Positive
Epstein-Barr virus early antigen
Negative
Epstein-Barr virus nuclear antigen
Negative
Antinuclear antibody (ANA) titer
1:40
Thyrotropin
Normal
The boy’s mother is very concerned about the ANA titer result because her husband’s sister was recently diagnosed with systemic lupus erythematosus. The boy’s symptoms are significantly improved. He has no significant medical history, and the results of a review of systems are negative. His vital signs and findings on a physical examination today are normal.
Of the following, the BEST next step in this boy’s management is to
A. advise that the test result is likely due to a viral infection B. obtain an antinuclear antibody profile evaluation C. refer him to a pediatric rheumatologist D. treat him with a 5-day course of corticosteroids
The adolescent in the vignette has a low positive antinuclear antibody (ANA) titer most likely due to his recent Epstein-Barr virus infection. Low ANA titers (<1:320) are usually not indicative of an autoimmune process, so he should not be referred to a rheumatologist at this time and should not have an ANA profile evaluation performed, unless he exhibits other specific signs of rheumatologic disease. He also does not need to receive corticosteroids for mononucleosis because his symptoms have significantly improved.
PREP Pearls
Up to one-third of healthy children may have a positive antinuclear antibody titer result.
Low antinuclear antibody titers (<1:320) are usually not indicative of an autoimmune process and may be due to viral illnesses, such as mononucleosis or parvovirus.
A 3-year-old girl is brought to the emergency department after drinking an unknown amount of mouthwash. The girl’s mother found her with an empty bottle of mouthwash in the bathroom. She is unsure how much had been in the bottle. The girl had 4 episodes of nonbloody, nonbilious vomiting prior to arriving at the emergency department. She has a temperature of 36.7°C, heart rate of 138 beats/min, respiratory rate of 12 breaths/min, and blood pressure of 108/66 mm Hg. She appears lethargic and is poorly responsive to painful stimuli. The remainder of the girl’s physical examination findings are unremarkable. Her airway, breathing, and circulation are stabilized.
Of the following, the BEST next step in this girl’s management is to
A. administer activated charcoal B. administer fomepizole C. order an arterial blood gas analysis D. order a bedside blood glucose level
The girl in the vignette has ingested an unknown amount of mouthwash, which contains ethanol (ethyl alcohol). In addition to beer, wine, and liquor, ethanol is found in a wide variety of household items including hand sanitizer, liquid cough and cold medications, cooking extracts (eg, vanilla), mouthwash, and perfumes/colognes. Because ethanol inhibits hepatic gluconeogenesis, children (especially young children) who ingest ethanol can become profoundly hypoglycemic. Therefore, the best next management step for the girl in the vignette is to obtain a bedside blood glucose level.
A pediatrician is called to an urgent cesarean delivery at 38 weeks’ gestation due to fetal bradycardia. The 36-year-old mother’s first prenatal visit was at 37 weeks’ gestation, when ultrasonography showed severe oligohydramnios; the fetal kidneys could not be visualized. The findings were discussed with the mother, and she was scheduled to see the maternal-fetal specialist but went into labor before the appointment. The clinician arrives for the delivery and prepares for resuscitation. There was no opportunity to talk to the mother before delivery about the fetus’ condition and plans for care.
Of the following, the BEST next management step after delivery will most likely be
A. administration of intravenous antibiotics B. endotracheal intubation for positive-pressure ventilation C. performance of renal ultrasonography D. umbilical line placement for intravenous glucose
Of the response choices, the best next management step after delivery of this neonate will most likely be endotracheal intubation to provide positive-pressure ventilation. The fetal ultrasonography findings are consistent with renal agenesis. Neonates born with bilateral renal agenesis have significant pulmonary hypoplasia resulting from severe oligohydramnios. Fetal urine contributes significantly to the amniotic fluid volume that is needed for fetal lung development. Immediate resuscitative efforts include positive-pressure ventilation through an endotracheal tube.
PREP Pearls
Fetal urine contributes significantly to amniotic fluid volume.
Amniotic fluid is necessary for fetal lung development.
Fetal renal agenesis results in severe oligohydramnios which leads to pulmonary hypoplasia.
A 16-year-old adolescent boy is seen for follow-up the morning after an emergency department visit for an asthma exacerbation. The adolescent has known asthma and allergies to grass and ragweed. Although prescribed daily, he takes his controller medication only as needed and has not felt the need to use any recently. Yesterday, he helped a friend mow a large field. About 15 minutes after starting to cut the tall grass he felt chest tightness and began to cough. He used his albuterol inhaler, which brought some relief, and took 2 to 4 puffs several times over the next 4 hours until the job was done. After going home and showering he felt better for a while. About 7 to 8 hours after he started cutting the grass he developed wheezing that was not fully relieved by his albuterol inhaler, prompting a visit to the emergency department. In the emergency department he was treated with oral corticosteroids and bronchodilators. Today, he feels much better, and his physical examination findings are normal.
Of the following, the BEST next step in this adolescent’s management is
A. an albuterol inhaler as needed and before known allergen exposure B. as-needed use of inhaled corticosteroid with an albuterol inhaler C. daily and as-needed use of a combined budesonide/formoterol inhaler D. daily use of inhaled tiotropium and formoterol
The adolescent in the vignette had a biphasic asthma response to allergen exposure, with early bronchospasm amenable to bronchodilator treatment followed hours later by a more intense airway inflammatory reaction responsive only to corticosteroids. This biphasic response is characteristic of IgE-mediated allergic asthma. The best next management step for this adolescent is daily and as-needed use of a combined budesonide/formoterol inhaler. A combined inhaled corticosteroid/long-acting β-agonist (ICS/LABA) inhaler used daily and additionally before allergen exposure prevents both early and late responses in those with IgE-mediated allergic asthma. Formoterol is an LABA appropriate for use in asthma management only when combined with an ICS. It is the only LABA with a short enough onset of action to provide acute relief, making it appropriate for as-needed administration (unlike salmeterol). Under the Same Maintenance And Rescue Therapy (SMART) paradigm from the 2020 National Asthma Education and Prevention Program guidelines, the combination of an ICS with formoterol as the LABA is appropriate for both daily maintenance and as-needed treatment.
PREP Pearls
IgE-mediated allergic asthma is characterized by a biphasic response including early bronchospasm and delayed airway inflammation.
The early response in IgE-mediated allergic asthma is amenable to prevention and treatment with bronchodilators, but corticosteroids are the most effective therapy for both prevention and treatment of the delayed inflammatory response.
Injectable anti-IgE monoclonal antibodies (eg, omalizumab) can prevent both the early and late phases of allergic asthma by binding circulating IgE and preventing its binding to allergens on mast cells.
A 14-year-old adolescent girl is seen in the office for a health supervision visit. Her menarche was 1 year ago; her cycles are irregular, with her last menstrual period occurring 5 weeks before this visit. After her first cycle, she did not have any bleeding for about 40 days; since then, her cycle intervals have ranged from 25 to 45 days. Her blood flow lasts for 4 to 6 days with the use of 2 to 3 pads per day. The adolescent is otherwise healthy with no history of medical problems. She is concerned about her irregular cycles. Her height is 165 cm (75th percentile) and weight is 50 kg (50th percentile); the percentiles are similar to those documented at her annual visit a year ago. Her physical examination findings are normal.
Of the following, the BEST explanation for this adolescent’s irregular menstrual cycles is
A. anorexia nervosa B. hypothyroidism C. immature hypothalamic-pituitary axis D. polycystic ovarian syndrome
It is common for adolescents to have irregular menstrual cycles, especially in the first 2 years after menarche. This phenomenon is due to immaturity of the hypothalamic-pituitary axis, which leads to anovulatory cycles. The interval between menstrual cycles can range from 21 to 45 days; 5% to 10% of girls have intervals shorter than 20 days or longer than 60 days. Prolonged intervals are most commonly seen between the first and second cycle. Bleeding normally occurs for 3 to 7 days. After the initial 2 years, most adolescents begin to have more regular cycles with intervals of 21 to 34 days, typical of adult women.
An 8-year-old boy with attention-deficit/hyperactivity disorder, combined type, is brought to the office for medication follow-up. His mother reports that the stimulant medication is working well, but she and his teacher are concerned because the boy’s grades have dropped and his frustration with school has increased since entering the third grade. He takes up to 2 hours to complete his homework. His mother tries to limit distractions by having the television off and sitting with him while he works, but it has not helped. The boy’s teacher allows extra time for him to complete his work, and he takes tests in a quiet office to limit distractions. He has been much less hyperactive and impulsive since starting the medication. The boy’s physical examination findings are unremarkable.
Of the following, the BEST next step in this boy’s management is to
A. increase the dose of his stimulant medication B. recommend a school evaluation for a learning disability C. recommend parent behavior-management training D. request a 504 plan through the school for additional accommodations
The boy in the vignette, with known attention-deficit/hyperactivity disorder (ADHD), is exhibiting signs of a learning disability. When he advanced to a higher grade in school, he began to have significant academic difficulty (poor grades, frustration with school, and prolonged time to complete homework despite behaviorally effective treatment with a stimulant medication). Common comorbidities in children with ADHD include learning disabilities, anxiety, depression, and oppositional defiant disorder. To assess the boy for a learning disability, an evaluation that includes both cognitive function and academic achievement should be performed.
A 3-month-old male infant born at term is admitted to the hospital with a 2-week history of fevers without a source.
Laboratory data are shown:
Laboratory test
Result
White blood cell count
1,500/µL (1.5 × 109/L)
Hemoglobin
8.0 g/dL (80 g/L)
Platelet count
50 × 103/μL (50 × 109/L)
Alanine aminotransferase
5,500 U/L
Aspartate aminotransferase
4,500 U/L
Serum bilirubin
6 mg/dL (102.6 µmol/L)
Serum triglycerides
500 mg/dL (5.7 mmol/L)
Erythrocyte sedimentation rate
150 mm/h
Serum ferritin
12,000 ng/mL (12,000 µg/L)
Interleukin 2 soluble receptor levels
2,000 pg/mL (reference range, 175.3-858.2 pg/mL)
Of the following, the MOST likely physical examination finding expected in this condition is
A. hearing loss B. snuffles C. splenomegaly D. vesicular rash
The infant in the vignette has prolonged fevers, cytopenia, evidence of inflammation, elevated transaminases, and elevated soluble interleukin 2 (IL-2) receptor levels. This clinical presentation is consistent with hemophagocytic lymphohistiocytosis (HLH). Although seen in all age groups, the highest incidence is in infants younger than 3 months. This condition can be familial or acquired. The most common infectious trigger of acquired HLH is Epstein-Barr virus (EBV), and the most common physical examination finding is splenomegaly (89% of cases).
6-year-old boy is brought to the office for school concerns. After an evaluation by the school his mother was told that he has an intellectual disability. She was surprised to hear this diagnosis as she thought the testing was “just for his reading.”
Review of the boy’s medical record reveals that he received early intervention services (speech and occupational therapy) beginning at age 24 months to address limited speech, difficulties with independent feeding, and fine motor skills. The boy’s mother decided to “take a break from therapies to let him catch up” after he aged out of early intervention at 3 years of age. During all subsequent health supervision visits, global impairments in speech, fine motor skills, problem solving, and adaptive skills were documented, and the boy’s mother was strongly advised to have him evaluated by the school when he entered prekindergarten.
The boy is not able to dress independently, is unable to tell a story, struggles with verbal instructions involving multiple steps, and prefers to play with younger children. He was toilet trained at age 5 years. The boy’s teacher raised academic concerns including his inability to identify letters or numbers, or write his name. Behavioral concerns at school include inattention, hyperactivity, frequent emotional outbursts, and rare aggressive behaviors.
The boy’s mother reports that her brother did not start talking until he was 5 years old, struggled with learning to read, required special education services, and did not graduate from high school. The boy’s mother does not want to “let the same thing happen” to her son. She plans to have him retained in the first grade because she feels that he is “not trying hard enough” and “only needs help with reading.”
Of the following, the finding that MOST supports the school’s diagnosis is the
A. family history of speech and learning difficulties B. history of inattention, hyperactivity, and externalizing behaviors C. receipt of speech therapy at an early age D. significant impairments in cognitive and adaptive functioning from an early age
The boy in the vignette has an intellectual disability. He has a history of developmental delays from a young age, and these impairments have persisted in all areas, including cognitive and adaptive functioning, thus supporting the diagnosis made with the formal school evaluation. The other response choices are not specifically associated with intellectual disability.
When a child has school performance concerns, information should be obtained regarding attainment of developmental milestones, social history, birth history, past medical history, and family history (family members with similar difficulties or concerns). For the boy in the vignette, it is unclear from the history provided if his uncle had a reading disability or intellectual disability.
A 4-day-old male infant is brought to the emergency department for evaluation of fast breathing and poor feeding. He was born at term via vaginal delivery and went home from the hospital the next day. His mother’s breast milk came in a couple of days ago, and he had been eating well until 8 hours before presentation. He latches well but seems to be breathing too quickly to suck and swallow. On physical examination, his heart rate is 170 beats/min, respiratory rate is 70 breaths/min, blood pressure is 70/40 mm Hg in the right arm, and oxygen saturation is 65% in room air measured in the right hand. He is tachypneic but not in distress. His lungs are clear to auscultation bilaterally, heart sounds are a normal S1 and a single S2 with a II/VI holosystolic murmur at the left sternal border, abdomen is benign, and capillary refill is brisk. A chest radiograph is obtained (Item Q36). boot shaped heart
Of the following, the BEST next management step for this neonate is to
A. initiate prostaglandin infusion B. intubate for impending respiratory failure C. perform echocardiography D. perform electrocardiography
The best next management step for the neonate in the vignette, who has tachypnea, desaturation, and poor feeding, is to initiate treatment with prostaglandin. His physical examination findings are significant for tachycardia, tachypnea, clear lungs, a single second heart sound, and a holosystolic murmur. His chest radiograph demonstrates a right aortic arch and a boot-shaped heart. These findings are concerning for congenital heart disease with insufficient pulmonary blood flow. Prostaglandin needs to be initiated urgently to open the closing ductus arteriosus to increase pulmonary blood flow and increase the oxygen saturation. Echocardiography and electrocardiography will be required to understand the details of his cardiac diagnosis, but they can be performed after the prostaglandin infusion is started. There is no indication for intubation of this child at this time.
PREP Pearls
Congenital cyanotic heart disease occurs when there is insufficient pulmonary blood flow, intracardiac mixing, or transposition physiology.
Prostaglandin infusion maintains patency of the ductus arteriosus. Higher doses are needed when the ductus has closed.
The side effects of prostaglandin infusion include apnea, fever, tachycardia, vasodilation, and hypotension.
A 14-year-old adolescent boy is seen in the emergency department with acute flaccid paralysis of the lower extremities, ascending numbness to the mid-trunk, urinary retention, and constipation. His vital signs are normal for age. On physical examination, his mental status and cranial nerve function are normal. He has normal upper extremity strength and reflexes. Strength in both lower extremities is 0/5, reflexes are brisk at the patella and ankle, and toes are upgoing to plantar stimulation. There is a sensory level at the mid-thorax below which he has diminished sensation to light touch and pinprick. The adolescent has normal coordination in his upper extremities. His lower extremity coordination and gait cannot be assessed due to his weakness. The remainder of his physical examination findings are unremarkable.
Magnetic resonance imaging (MRI) of the spinal cord shows an extensive longitudinal T2 hyperintensity extending from T3 to the conus with no enhancement. Brain MRI is normal. Lumbar puncture is performed, and cerebrospinal fluid analysis shows an elevated white blood cell count of 100 cells/μL with a lymphocytic predominance. Antibody tests are ordered.
The adolescent is admitted to the intensive care unit and treated with intravenous methylprednisolone daily for 5 days with minimal improvement. He is subsequently treated with plasma exchange. Rehabilitation services are engaged early in his disease course.
Of the following, the BEST additional supportive measure(s) in the acute management of this adolescent is/are
A. anticoagulation with therapeutic intravenous heparin infusion B. bladder ultrasonography with intermittent catheterization C. permissive hypertension with intravenous phenylephrine D. prophylactic neuropathic pain management with oral gabapentin
The adolescent in the vignette has acute transverse myelitis, a disorder of the spinal cord characterized by acute onset of motor, sensory, and sphincter deficits with a range of underlying etiologies. Urinary retention is a common complication, which increases the risk of urinary tract infection. Therefore, of the response choices, the best additional supportive measures in this adolescent’s acute management are bladder ultrasonography and intermittent catheterization.
PREP Pearls
Acute transverse myelitis is a disorder of the spinal cord characterized by acute onset of motor, sensory, and sphincter deficits with a range of underlying etiologies, most commonly inflammatory or autoimmune. Radicular or back pain may precede the acute onset of neurologic symptoms.
Urgent evaluation of suspected transverse myelitis should include spine magnetic resonance imaging (MRI) to exclude cord compression. Most commonly, imaging shows centrally located hyperintensity spanning multiple contiguous vertebral levels; up to 50% have normal findings on MRI.
In addition to disease-specific treatment of transverse myelitis with high-dose intravenous corticosteroids or plasma-exchange, supportive management includes bowel and bladder care, deep vein thrombosis prophylaxis, decubitus ulcer prevention, and treatment of autonomic instability.
An 11-year-old girl is being evaluated for an injury to her left ankle that she sustained while playing basketball. On the day before her visit to the clinic, she inverted her ankle when she landed awkwardly from a jump. On physical examination, she has moderate swelling over the lateral aspect of her left ankle. There is tenderness to palpation over the anterior aspect of the distal lateral malleolus and just anterior and inferior to this spot. She has mild laxity with anterior drawer testing. She has an antalgic gait and reports mild pain when walking.
Of the following, the BEST next step in this girl’s management is
A. cast immobilization B. gradual return to basketball as tolerated C. limiting weight-bearing with the use of crutches D. use of an air stirrup splint
The most likely diagnosis for the girl in the vignette is a sprain of the anterior talofibular ligament (ATFL). Item C39A illustrates the position of the ATFL, calcaneofibular ligament (CFL) and posterior talofibular ligament (PTFL) on the lateral aspect of the ankle. She had an ankle inversion injury and has tenderness over the anterior aspect of the distal lateral malleolus, where the ligament inserts, and over the ligament itself.
Following an ankle sprain, recovery is faster with early mobilization of the ankle. A stirrup style ankle brace (Item C39B) permits dorsiflexion and plantarflexion at the ankle, which allows for a more normal gait, while limiting eversion and inversion.
Use of crutches with limited weight-bearing is rarely indicated. Physicians should encourage early mobilization with the use of a less restrictive brace when injured athletes can tolerate it.
PREP Pearls
Early mobilization after an ankle sprain leads to a faster recovery.
The Ottawa ankle rules for obtaining radiographs after an ankle injury have been validated in children.
A male infant is seen for a routine health supervision visit. His parents have no concerns. He sits unassisted on the examination table. He becomes excited when his mother hands him a favorite toy, reaches up to shake it, and places it in his mouth. While the examiner is talking to his mother, the infant babbles and smiles when his mother looks at him.
Of the following, this infant’s age is MOST likely
A. 3 months B. 4 months C. 6 months D. 9 months
The infant in the vignette is exhibiting developmental milestones most consistent with a 6-month-old. He responds to his mother, babbles, sits without support, and brings an object to his mouth.
PREP Pearls
By 6 months of age, infants should be able to sit independently or with their own hands for support.
By 6 months of age, infants should babble with vowel sounds and begin to add consonant sounds.
By 6 months of age, infants should be able to cross midline to reach for objects.
PerformanceConfidenceAssessment History
Search
All Questions
Question View:
All
41 of 269Print Add Bookmark
ASSESSMENT PROGRESS:Total Questions: 269 Questions Answered: 40 Correct Answers: 26
Question 41
A 6-year-old boy is brought to the emergency department with tea-colored urine for 1 day. He has had no flank pain, burning, or passage of blood clots with urination. He had a sore throat 4 weeks ago. On physical examination, the boy’s heart rate is 110 beats/min, respiratory rate is 18 breaths/min, and blood pressure is 136/86 mm Hg. He has mild puffiness over his eyes; the rest of the physical examination findings are unremarkable.
Results of a urinalysis with microscopy are shown:
Urine
Result
Specific gravity
1.030
Leukocyte esterase
Negative
Nitrite
Negative
Blood
3+
Protein
3+
Red blood cells
> 100/HPF
White blood cells
< 5/HPF
Of the following, the MOST likely diagnosis for this boy is
A. cystitis B. glomerulonephritis C. nephrolithiasis D. rhabdomyolysis
The child in the vignette had a sore throat 4 weeks before presentation with macroscopic hematuria (tea-colored urine without blood clots), hypertension, and edema. Urinalysis with microscopy shows hematuria and proteinuria favoring a diagnosis of acute poststreptococcal glomerulonephritis (PSGN).
PREP Pearls
Brown, tea, or cola-colored urine without blood clots favors a glomerular etiology of macroscopic hematuria.
Bright red, red, or pink urine with blood clots is suggestive of hematuria originating from the urinary collecting system.
A urine that is dipstick test–positive for blood without red blood cells on urine microscopy indicates myoglobinuria (eg, rhabdomyolysis) or hemoglobinuria (eg, intravascular hemolysis).
ABP Content Specifications(s)
Formulate a differential diagnosis of gross hematuria
Plan the appropriate clinical evaluation of gross hematuria
Recognize the disorders associated with hematuria
A 3-day-old male infant is seen in the emergency department for decreased feeding, vomiting, and lethargy. He has been taking 2 ounces of formula every 2 hours but vomited his last feed and has been difficult to arouse. His vital signs include a blood pressure of 68/40 mm Hg, temperature of 37.3°C, heart rate of 170 beats/min, and respiratory rate of 80 breaths/min. His newborn metabolic screen results are pending.
Laboratory data are shown:
Laboratory Test
Result
Glucose
38 mg/dL (2.1 mmol/L)
Ammonia
35 µg/dL (25 µmol/L)
Carbon dioxide
16 mEq/L (16 mmol/L)
Lactic acid
3 mmol/L
Urine ketones
Positive
Urine reducing substances
Positive
Serum total bilirubin
14 mg/dL (239.4 µg/dL)
Aspartate aminotransferase
90 U/L
Alanine aminotransferase
60 U/L
White blood cell count
8,000/µL (8.0 × 109/L)
C-reactive protein
0.2 mg/dL (2 mg/L)
Of the following, the neonate’s MOST likely diagnosis is
A. fatty acid oxidation disorder B. galactosemia C. organic acidemia D. urea cycle defect
The neonate in the vignette has galactosemia as evidenced by hypoglycemia, hyperbilirubinemia, elevated liver enzymes, and the presence of reducing substances in the urine. The diagnosis of classic galactosemia is made through newborn screening or after the onset of clinical signs and symptoms consistent with the condition. In an untreated neonate, classic galactosemia presents with a life-threatening illness that can involve the liver, brain, and kidney. Neonates are normal at birth; symptoms develop after ingestion of breast milk or standard formula, which leads to accumulation of galactose and its by-products (galactose-1-phosphate, galactitol, and galactonate) in the body.
Urea cycle defects present in the neonatal period with hyperammonemia and respiratory alkalosis. Common urea cycle defects include ornithine transcarbamylase deficiency, citrullinemia, and argininosuccinic aciduria.
A 5-week-old male infant is seen in the clinic for a routine follow-up visit. He has had worsening emesis over the past 2 weeks that occurs after all feedings and is yellow in color. He has had no fever or respiratory symptoms, but he is progressively becoming more fussy. The infant has lost 110 g since his last visit 2 weeks ago. He is too fussy to obtain an accurate abdominal examination.
Of the following, this infant’s diagnostic evaluation would MOST likely show
A. abnormal upper gastrointestinal series with small bowel follow through B. elevated total body potassium C. esophageal erosion D. hypochloremic metabolic alkalosis
The infant in the vignette has a clinical presentation that is consistent with a diagnosis of hypertrophic pyloric stenosis (HPS). On abdominal examination of an infant with pyloric stenosis, an “olive-sized” epigastric mass is palpable in 60% to 80% of cases, and peristaltic waves may be visible.
PREP Pearls
Abdominal examination of an infant with hypertrophic pyloric stenosis may demonstrate a palpable epigastric “olive” or visible peristaltic waves.
Hypertrophic pyloric stenosis causes a hypochloremic metabolic alkalosis.
Abdominal ultrasonography is the imaging study of choice to diagnose hypertrophic pyloric stenosis.
ABP Content Specifications(s)
Recognize the acid-base changes associated with pyloric stenosis, and manage appropriately
Recognize the clinical features associated with pyloric stenosis, and manage appropriately
A 2-week-old male infant, born at 35 weeks’ gestation, is seen at the office for a weight check. He is breastfeeding well with normal urination and stooling. He has gained 5 ounces since birth. On physical examination, the neonate is comfortable in room air, with a respiratory rate of 40 breaths/min and a temperature of 37.5°C. There is bilateral watery eye discharge, palpebral and bulbar injection, and mild eyelid swelling. The neonate received eye prophylaxis at birth. The conjunctiva and nasopharynx are swabbed for bacterial culture and a polymerase chain reaction test for gonorrhea and chlamydia.
Of the following, the BEST next step in management of this neonate is
A. azithromycin eye ointment B. azithromycin oral suspension C. ceftriaxone, intramuscular D. external digital massage of the tear ducts
The neonate in the vignette has findings consistent with chlamydial conjunctivitis. Neonatal chlamydial infection is acquired during passage through the infected birth canal. Early in the course, chlamydial conjunctivitis can be watery, and later it becomes mucopurulent. There may be swelling of the eyelids and chemosis (swelling and redness of conjunctiva). Eye prophylaxis with erythromycin ointment given at birth is not effective in preventing chlamydial infection. Oral azithromycin or oral erythromycin is the treatment of choice for both chlamydial conjunctivitis and pneumonia in neonates.
A single dose of intramuscular ceftriaxone, 50 mg/kg, is recommended for treatment of gonococcal ophthalmia, but is not effective for treatment of a chlamydial infection. Although increased tearing and matting of the eyelashes is commonly seen with nasolacrimal duct obstruction, conjunctival erythema, edema, and swelling are not common. The presence of eyelid swelling and chemosis should prompt further investigation and consideration of treatment for neonatal conjunctivitis.
PREP Pearls
Chlamydia, an obligate intracellular bacterium, causes the most common sexually transmitted notifiable infection in the United States.
Neonates exposed to Chlamydia through an infected birth canal develop conjunctivitis typically 5 to 14 days after birth; chlamydial pneumonia occurs between 3 and 12 weeks of age.
The recommended treatment for any form of neonatal chlamydial infection is oral azithromycin or erythromycin
A 6-week-old infant with no significant birth or past medical history is brought to the emergency department by ambulance for decreased responsiveness. He appears lethargic, pale, and cyanotic with poor respiratory effort. His capillary refill time is greater than 5 seconds. His vital signs include a temperature of 36.5°C, heart rate of 198 beats/min, respiratory rate of 8 breaths/min, blood pressure of 65/42 mm Hg, and oxygen saturation of 86% in room air.
Of the following, the BEST next step in this infant’s management is to
A. administer a 20 mL/kg normal saline fluid bolus B. administer broad-spectrum antibiotics intravenously C. begin bag-valve mask ventilation and prepare for intubation D. obtain a point-of-care glucose level
The infant in the vignette is displaying altered mental status, for which the initial management is the ABCs (Airway, Breathing, Circulation) of resuscitation. Assessment and stabilization of the airway is the first step, followed by stabilization of breathing and circulation. The infant in the vignette is cyanotic with poor respiratory effort and a respiratory rate of 8 breaths/min; thus, bag-valve mask ventilation and preparing for intubation is the best next management step. Although obtaining a point-of-care glucose level and administration of intravenous fluids and broad-spectrum antibiotics will be important steps in the infant’s care, addressing the airway and breathing concerns must take priority.
PREP Pearls
The initial management of altered mental status is the ABCs (Airway, Breathing, Circulation) of resuscitation.
The signs and symptoms of altered mental status in a child can range from subtle to overt and vary depending upon the age of presentation.
The mnemonic AEIOU TIPS is helpful for remembering the broad differential diagnosis for causes of altered mental status.
A 3-day-old neonate is seen in the office after his newborn screening test showed a thyroxine (T4) level of 1.2 µg/dL (15.5 nmol/L) (reference range, ≥6.5 µg/dL [≥83.7 nmol/L]. The reflex thyroid-stimulating hormone (TSH) level is 2.5 mIU/L (reference range, ≤20 mIU/L ). The neonate was born at term via uncomplicated labor and delivery and had no problems in the newborn nursery. He is exclusively breastfed, and his weight today is 3% below birth weight. The neonate appears vigorous and has mild jaundice. He does not have a goiter or umbilical hernia. The remainder of his physical examination findings are normal.
Of the following, the BEST next step in the management of this neonate is to
A. begin levothyroxine therapy B. obtain thyroid-stimulating hormone and free thyroxine levels C. order magnetic resonance imaging of the pituitary gland D. repeat the newborn screening test when the neonate is 5 to 7 days old
The neonate in the vignette has an abnormal newborn screening test with a low thyroxine (T4) and normal thyroid-stimulating hormone (TSH) level. Based on the neonate’s physical examination findings, the next best step in management is to obtain confirmatory serum TSH and free thyroxine (FT4) levels. The most likely diagnosis for this healthy male neonate is congenital thyroxine-binding globulin (TBG) deficiency, a benign X-linked condition. Confirmatory testing would demonstrate normal TSH and FT4 levels.
Neonates who have newborn screening performed too soon after birth may have elevated TSH levels, given the expected surge of thyroid hormones that occurs after delivery. In such a case, repeating the newborn screening test may be recommended. However, this would not be true for the neonate in the vignette, as his pattern of thyroid function results is not consistent with the newborn screening test being drawn too soon after birth.
PREP Pearls
Neonates with an abnormal thyroid newborn screening test result should be evaluated promptly with a physical examination and confirmatory serum thyroid-stimulating hormone and free thyroxine testing.
A newborn screening test result showing a low thyroxine and normal thyroid-stimulating hormone level is consistent with thyroxine-binding globulin (TBG) deficiency or, less frequently, central hypothyroidism. The confirmatory serum free thyroxine level distinguishes these conditions (normal in TBG deficiency and low in central hypothyroidism).
Thyroxine-binding globulin deficiency is a benign, X-linked condition that does not require treatment.
A 38-year-old, gravida 7, para 5 woman who is 38 weeks’ pregnant is brought to the labor and delivery unit for severe abdominal pain. She has a history of diabetes and hypertension. She received limited prenatal care. Ultrasonography 1 month ago showed that the placenta was abnormally placed. The woman’s blood pressure is 230/140 mm Hg. The health care team is preparing to perform an urgent cesarean section because of the fetal heart tracing shown in Item Q49. Before delivery, the team is planning for the neonate’s potential management needs beyond standard resuscitation.
Of the following, in the delivery room this neonate is MOST likely to require
A. blood transfusion B. chest radiography C. intravenous calcium gluconate D. surfactant administration
The clinical scenario described in the vignette is suggestive of a placental abruption with fetal blood loss. The neonate in the vignette is most likely to require a blood transfusion of uncrossmatched O-negative blood.
PREP Pearls
Most neonates transition to the extrauterine environment without difficulty. Five percent require some form of resuscitation.
Anticipation and planning are key components of neonatal resuscitation.
Ventilation is the most important step in neonatal resuscitation.
A 7-year-old boy is seen in the emergency department for evaluation of a 2-day history of abdominal pain, watery diarrhea, malaise, and loss of appetite. Five days ago, he visited a family-owned farm with his siblings. He swam in a chlorinated pool located on the farm and had exposure to livestock, including calves. Other members of the family are also ill with diarrhea. The boy’s temperature is 37.4°C; his vital signs are normal for age. His physical examination findings are unremarkable except for dry oral mucous membranes.
Of the following, the MOST likely cause of this boy’s illness is
A. Clostridium perfringens B. Cryptosporidium parvum C. enteropathogenic Escherichia coli D. norovirus
The boy in the vignette had onset of watery diarrhea 3 days after exposure to treated (ie, chlorinated, filtrated) recreational water. This presentation is most suggestive of Cryptosporidium parvum gastroenteritis. The incubation period of cryptosporidiosis ranges from 3 to 14 days.
Recreational water-associated outbreaks can be caused by a variety of pathogens including Escherichia coli O157:H7, Shigella, norovirus, and Giardia. Sporadic outbreaks of norovirus gastroenteritis are common. However, the incubation period of norovirus infection is short (12 to 24 hours) and characterized by sudden onset of vomiting followed by watery diarrhea.
PREP Pearls
Cryptosporidium is a leading cause of diarrheal outbreaks associated with exposure to chlorinated recreational water (eg, swimming pools, waterslides).
Fecal-oral transmission of Cryptosporidium infection occurs via ingestion of oocysts excreted by infected hosts.
Manifestations of cryptosporidiosis include non-bloody, watery diarrhea with abdominal pain, nausea, vomiting, low-grade fever, anorexia, and weight loss; asymptomatic gastrointestinal infection can occur resulting in failure to thrive.
An 8-month-old boy is evaluated for a 3-day history of worsening cough, nasal congestion, and low-grade fever. His mother reports that he is irritable, has difficulty breathing, and is eating and drinking less than usual. He has good urine output.
On physical examination, the infant is ill-appearing and fussy but consolable. His temperature is 38.5°C, heart rate is 102 beats/min, respiratory rate is 55 breaths/min, and oxygen saturation is 95% in room air. He has mild intercostal retractions and nasal flaring. Breath sounds are coarse throughout with crackles in the right base extending into the right axilla; no wheezing is heard. The remainder of the physical examination findings are unremarkable.
Of the following, the BEST next step in this infant’s management is to
A. administer antibiotics B. obtain computed tomography scan of the chest C. perform rapid intubation and start mechanical ventilation D. recommend he return for reevaluation tomorrow
The infant in the vignette with fever, tachypnea, increased work of breathing, and focal crackles on physical examination most likely has community-acquired pneumonia (CAP). Although most CAP in children is caused by viruses, this infant’s focal findings and worsening symptoms make a bacterial etiology more likely. Based on the 2011 Infectious Diseases Society of America (IDSA) guidelines for management of CAP, administration of antibiotics is the best next intervention for this infant with likely bacterial pneumonia.
The infant in the vignette appears ill and meets the IDSA guideline criteria for respiratory distress (Item C52A). Admission to the hospital should be considered, and chest radiography would be appropriate; however chest computed tomography is not indicated as a first-line imaging study. Although the infant is in mild distress, he has normal oxygen saturation and does not show signs of imminent respiratory failure; therefore, there is no indication for intubation and mechanical ventilation. However, given his level of distress and the concern for bacterial pneumonia, he does need intervention beyond observation and reevaluation. An important consideration regarding his need for hospital admission is his risk for dehydration, as his mother reports poor oral intake.
PREP Pearls
Normal respiratory rate varies widely across ages; infants show greater variability than older children.
Respiratory rate is a sensitive indicator of illness; abnormal sleeping respiratory rate is the best measure of true dysfunction.
Any child with unexplained tachypnea or hypopnea should undergo careful evaluation for underlying illness.
A 6-month-old infant, born at 27 weeks’ gestation, with grade 2 vesicoureteral reflux, gastroesophageal reflux, and chronic lung disease is seen in the emergency department for lethargy. His daily medications include fluticasone, furosemide, lansoprazole, and nitrofurantoin. His temperature is 36.8°C, heart rate is 120 beats/min, respiratory rate is 30 breaths/min, blood pressure is 85/65 mm Hg, and oxygen saturation is 99% on his home oxygen support of 0.5 L/min via nasal cannula. The infant’s anterior fontanelle is slightly sunken, and his mucous membranes are moist and pink. On lung auscultation, fine rhonchi are heard throughout without wheezing or rales. There are no retractions or other signs of increased work of breathing. The remainder of his physical examination findings are normal.
Laboratory data are shown:
Laboratory Test
Result
Sodium
130 mEq/L (130 mmol/L)
Potassium
2.8 mEq/L (2.8 mmol/L)
Chloride
89 mEq/L (89 mmol/L)
Carbon dioxide
38 mEq/L (38 mmol/L)
Blood urea nitrogen
10 mg/dL (3.6 mmol/L)
Creatinine
0.3 mg/dL (26.5 µmol/L)
Of the following, the medication MOST likely to have led to this infant’s findings is
A. fluticasone B. furosemide C. lansoprazole D. nitrofurantoin
The infant in the vignette, with chronic lung disease related to prematurity, has signs of mild dehydration (mildly sunken anterior fontanelle), and electrolyte abnormalities including hyponatremia, hypokalemia, hypochloremia, and an elevated bicarbonate level. These findings are consistent with a contraction metabolic alkalosis induced by chronic diuretic therapy. Thus, of the response choices, furosemide (used to control excess fluid in the lungs) is the most likely medication to have contributed to this infant’s condition.
Furosemide is a loop diuretic that inhibits reabsorption of sodium and chloride in the ascending loop of Henle and the proximal and distal renal tubules by interfering with the chloride-binding co-transport system. Both natriuresis (sodium loss) and diuresis (water loss) result. Diuresis decreases extracellular fluid volume and concentrates extracellular serum bicarbonate, contributing to metabolic alkalosis. Historically, this process was known as “contraction alkalosis,” and the primary mechanism was thought to be due to decreased fluid volume and the resulting concentration of serum bicarbonate. However, newer studies have demonstrated that chloride plays a major role in the development of metabolic alkalosis, and some have suggested that “chloride depletion alkalosis” replace the term “contraction alkalosis” to more accurately reflect the underlying mechanism.
PREP Pearls
Metabolic alkalosis is often caused by hypochloremia.
Infants and children on chronic diuretic therapy with loop diuretics (eg, furosemide) are at risk for developing a chloride depletion alkalosis.
Bartter syndrome can cause hypokalemic, hypochloremic, metabolic alkalosis; this diagnosis should be suspected in infants with hyponatremia, hypokalemia, failure to thrive, and dehydration.
greatest risk for perinatal transmission of hep b
Perinatal hepatitis B transmission occurs at the time of labor and delivery. The risk of transmission to the newborn is determined based on the presence of maternal hepatitis B surface antigen (HBsAg) and hepatitis B e antigen (HBeAg). The presence of HBsAg implies either an acute (<6 months) or chronic (≥6 months) infection in the mother. The presence of HBeAg indicates a high level of transmissibility. For a mother positive for both HBsAg and HBeAg, the risk of perinatal transmission is as high as 90%. For a mother that is only HBsAg positive, the risk is around 30%. Other risk factors associated with high transmissibility include a high viral DNA level >2,000 IU/mL, age <25 and <3 doses of maternal hepatitis B vaccine. Detection of hepatitis B surface antibody (>10 mIU/ML) indicates disease immunity, either from immunization or natural infection. Detection of hepatitis B e antibody along with hepatitis B surface antibody indicates past infection.
A 12-year-old girl is seen in the clinic for evaluation of a 3-month history of abdominal pain and diarrhea. She has 5 to 8 loose, malodorous, floating, greasy-appearing stools daily. She lost 5 kg unintentionally during the 3-month period. Physical examination reveals a weight of 36.3 kg (24th percentile for age), height of 150 cm (44th percentile for age), and a body mass index of 16 kg/m2 (19th percentile for age). She appears pale. Her abdomen is distended but soft and nontender. The remainder of the girl’s physical examination findings are unremarkable.
Laboratory data are shown:
Laboratory Test
Result
Stool
pH
7.0
Occult blood
Negative
Reducing substances
Negative
Ova and parasite
Negative
Bacterial pathogen testing
Negative
Blood
Hemoglobin
10.1 g/dL (101 g/L)
Erythrocyte sedimentation rate
2 mm/h
25-Hydroxyvitamin D
9 ng/mL (22.46 nmol/L)
Albumin
4.3 g/dL (43 g/L)
Of the following, the BEST treatment for this girl is
A. albumin 25% infusion B. gluten-free diet C. lactase enzyme replacement therapy D. low-sucrose diet
The girl in the vignette has evidence of increased intestinal fat losses (steatorrhea and fat-soluble vitamin D deficiency), resulting from fat maldigestion and/or fat malabsorption. Of the response choices, a gluten-free diet is the only treatment for a fat malabsorption disorder.
On the basis of the girl’s laboratory and physical examination findings, a diagnosis of celiac disease was suspected. Further testing revealed an elevated tissue transglutaminase IgA antibody level, and findings on endoscopy with biopsy confirmed the diagnosis. A gluten-free diet is the treatment for celiac disease. Albumin infusions are used to treat symptomatic hypoalbuminemia caused by severe protein-losing enteropathy. The albumin level of the girl in the vignette is within the reference range, and an albumin infusion would not address her fat malabsorption and steatorrhea. Lactase enzyme replacement therapy can be used to treat diarrhea that results from malabsorption of lactose, and a low-sugar diet may resolve diarrhea caused by malabsorption of sucrose, but neither treatment would decrease this girl’s fat malabsorption and steatorrhea.
PREP Pearls
Steatorrhea (Increased fecal fat) occurs when ingested fat is maldigested and/or malabsorbed.
Complications of fat maldigestion or malabsorption include diarrhea, abdominal pain, diaper dermatitis in infants and diapered children, weight loss or poor weight gain, and fat-soluble vitamin deficiencies.
Treatment of steatorrhea includes identifying and managing the underlying cause while supporting nutrition and growth through adequate caloric intake and fat-soluble vitamin supplementation.
A 4-year-old boy with trisomy 21 is seen in the emergency department with a femur fracture. His mother brought him to the emergency department because he would not walk when he awoke from his nap. There is no reported history of trauma. The boy’s past medical history is significant for an atrial septal defect that was repaired 9 months ago. He has an individualized education program and receives speech therapy through the school district twice weekly. He is incontinent of urine and stool. The boy lives with his mother and 2 older developmentally typical siblings. His mother states that she is unemployed because she must care for the boy during the day. The boy’s grandparents sometimes help with childcare. The family receives public assistance and has Medicaid insurance. Further evaluation in the emergency department reveals multiple healed rib fractures.
Of the following, the MOST significant risk factor for this boy’s injuries is
A. family need for public assistance B. male sex C. single-parent household D. trisomy 21
The boy in the vignette has injuries consistent with non-accidental trauma (NAT) or physical abuse (unexplained femur fracture, multiple healed rib fractures). Having trisomy 21 is his most significant risk factor for experiencing physical abuse. Children with developmental disabilities and chronic medical conditions are at increased risk for abuse. One study (Paul) found that children with disabilities are nearly 3 times more likely to be physically abused than children without a disability. Children with mild cognitive disabilities and without motor disabilities were found to be at the highest risk.
A 12-month-old, full-term male infant is being evaluated for gross motor delay. His mother first became concerned when he was 7 months of age because he was not yet rolling over. At that time laboratory testing demonstrated a serum creatine kinase level of 50 U/L and a thyroid-stimulating hormone level of 1.1 mIU/L. Newborn screening results were re-verified as normal. The infant was enrolled in physical therapy. He rolled (supine to prone and prone to supine) at age 9 months and was able to sit with support at 10 months of age. At age 12 months, he cannot sit independently. He reaches for toys with both hands, transfers objects, and is beginning to use 2 fingers in a pincer grasp to pick up cereal. He smiles, laughs, and babbles with multiple consonants. He enjoys playing peek-a-boo and with toys that he causes to light up and play music. There has been no developmental regression.
On physical examination, the infant’s vital signs are normal. His head circumference is 46 cm (41st percentile for age). On neurological examination, he is alert and playful with age-appropriate stranger anxiety. His cranial nerve findings are normal. He has truncal hypotonia with head lag when pulled to sit, slippage at the shoulders on vertical suspension, and mild appendicular hypotonia in all 4 extremities. His strength appears normal and symmetric. When placed in a supported sit, he has slight titubation of the head. Deep tendon reflexes are 2+ in the biceps, brachioradialis, and triceps, 3 at the patella bilaterally, and 2+ at the achilles bilaterally. His toes are downgoing to plantar stimulation. He reaches for toys without dysmetria/ataxia. The remainder of the infant’s physical examination findings are normal.
Of the following, the BEST next step in this infant’s evaluation is
A. brain magnetic resonance imaging B. electromyography C. nerve conduction studies D. survival motor neuron 1 gene testing
The infant in the vignette has an isolated gross motor delay in the setting of predominantly axial hypotonia with preserved reflexes and strength. These examination findings are suggestive of a central etiology for his hypotonia. Magnetic resonance imaging (MRI) of the brain would be an appropriate next step in his diagnostic evaluation to evaluate for cerebral malformations or neuroimaging markers of specific genetic and/or metabolic disorders. The other response choices would be more appropriate when testing for an infant with findings suggestive of hypotonia secondary to a peripheral etiology. In this age group, however, central etiologies are more common than peripheral.
The initial diagnostic testing for infants with hypotonia suspected to have a peripheral etiology is focused on time-sensitive and treatable conditions. Evaluation usually includes a serum creatinine kinase (CK) level, SMN1 gene deletion testing, and thyroid studies to evaluate for an underlying myopathy, spinal muscular atrophy, or thyroid disease. If the results of initial laboratory testing are negative, electromyography and nerve conduction studies should be performed for more precise localization within the peripheral nervous system prior to proceeding to targeted disease tests or muscle biopsy.
PREP Pearls
The signs and symptoms accompanying hypotonia provide important clues for accurate localization, which can encompass any part of the nervous system.
In infants, central etiologies of hypotonia are more common than peripheral.
Tone, the resistance of the muscle to passive range of motion/stretch, is distinct from strength, the amount of force a muscle group can generate. The presence or absence of weakness can be a helpful localization indicator.
A 14-year-old adolescent boy is seen for a preparticipation physical examination prior to his high school soccer season. The boy’s medical history is significant for a seizure 2 years ago that occurred during a middle school basketball game. His family history is remarkable for a sibling who died of sudden infant death syndrome and his mother and maternal uncle who have type 1 diabetes mellitus. The adolescent’s physical examination findings are normal.
Of the following, the BEST next management step for this boy is
A. clearance for soccer participation B. referral to a cardiologist C. referral to a endocrinologist D. referral to a neurologist
The boy in the vignette had an apparent seizure while exercising. Myoclonic activity associated with an arrhythmia can be mistaken for seizure activity. Seizures can also be associated with dysrhythmia events and can be the initial presenting sign of an underlying channelopathy, such as long QT syndrome. The boy had a sibling who died of sudden infant death syndrome, which can also be the sentinel event for a child with a congenital arrhythmia syndrome. The boy’s personal and family history are very concerning for a genetic heart condition, and he should be evaluated by cardiology prior to clearing him for sports participation.
PREP Pearls
A history of unexplained seizure activity could signify arrhythmogenic heart disease.
A child or adolescent with a well-controlled seizure disorder should be cleared for participation in most sports without qualification.
A 3-month-old infant is seen for a weight check. Her weight today is 5 kg, which is the same as her weight at an appointment 2 weeks ago. She is scheduled for surgical repair of a large ventricular septal defect at the end of the month and is on a high-dose diuretic regimen. She has persistent comfortable tachypnea and is otherwise asymptomatic. Her diet consists of expressed breast milk by bottle. Attempts at increasing the volume of feedings have failed secondary to disinterest from the infant.
Of the following, the BEST next dietary intervention for this infant is to
A. add electrolyte solution to aid in hydration B. change to a 30-kcal/oz formula C. fortify the breast milk D. limit feeding time to 20 minutes
The infant in the vignette has a large ventricular septal defect with increased pulmonary blood flow, leading to pulmonary edema and tachypnea. Her symptoms are managed with high-dose diuretics to decrease the pulmonary edema. Ultimately, she needs to undergo surgical repair of this lesion. Until then, the pediatrician and pediatric cardiologist can improve her surgical candidacy by maintaining appropriate weight gain. Fortifying the expressed breast milk is the best option to limit fluid intake while maximizing calories. Neither adding an electrolyte solution for hydration nor limiting feeding time to minimize energy use will provide the needed additional calories. Changing the infant’s diet to a 30-kcal/oz formula may result in diarrhea and excessive stool losses.
PREP Pearls
Children with heart disease have increased caloric needs and often require limitation of fluid intake.
Increased caloric density formula or fortified breast milk is often needed for appropriate weight gain in infants with heart disease.
A multidisciplinary approach is needed to optimize nutrition and growth for many children with heart disease.
A 15-year-old adolescent boy is evaluated in the office for right flank pain and bright red urine with blood clots for 1 day. His pain is intermittent and colicky, and is radiating to the right lower abdomen. He has no dysuria, urgency, or fever. The boy’s heart rate is 120 beats/min, respiratory rate is 16 breaths/min, and blood pressure is 110/70 mm Hg. His weight is greater than the 95th percentile, and his height is at the 50th percentile for age. He has right costovertebral angle tenderness. The remainder of his physical examination findings are unremarkable.
A urinalysis with microscopy is shown:
Laboratory Test
Result
Specific gravity
1.030
Leukocyte esterase
Positive
Nitrite
Negative
Blood
3+
Protein
Negative
Red blood cells
> 100/HPF
White blood cells
10-20/HPF
Of the following, the BEST next step in confirming this adolescent’s suspected diagnosis is
A. abdominal computed tomography with contrast B. abdominal magnetic resonance imaging without contrast C. plain abdominal radiography D. renal and bladder ultrasonography
The adolescent boy in the vignette has symptoms consistent with renal colic, macroscopic hematuria with blood clots, and right costovertebral tenderness, favoring a diagnosis of nephrolithiasis. Renal ultrasonography is the first-line imaging modality recommended to detect nephrolithiasis. Advantages of ultrasonography over the other options listed include the avoidance of exposure to ionizing radiation and the potential to detect obstruction in the urinary system. A noncontrast spiral computed tomography (CT) scan is the most sensitive test to diagnose nephrolithiasis, especially in symptomatic patients, however, the significant radiation exposure makes this a less optimal choice. A CT scan with contrast is not commonly recommended as the contrast may decrease the sensitivity for stone detection and extend the time required to perform the test. Renal calculi are often not detected on magnetic resonance imaging. Abdominal radiography has a lower sensitivity and specificity than does ultrasonography and detects only radio-opaque renal stones.
PREP Pearls
Factors that increase the risk for nephrolithiasis include decreased urine volume, increased solute excretion (calcium, uric acid), and decreased levels of stone inhibitors (citrate, magnesium).
Renal and bladder ultrasonography is the first-line imaging modality recommended to diagnose nephrolithiasis in children.
A noncontrast spiral computed tomography scan is the most sensitive test to diagnose nephrolithiasis in symptomatic children.
A 12-year-old girl is seen for a health supervision visit. Developmental milestones were delayed for speech, and she received speech therapy from age 2 to 4 years. She is in a 7th-grade, regular education classroom with an individualized education plan for reading and comprehension. Menarche was achieved at age 11 years. Physical examination findings are unremarkable except that she is a tall girl. Family history is noncontributory.
Of the following, the MOST likely diagnosis for this girl is
A. Klinefelter syndrome B. Marfan syndrome C. trisomy X D. Turner syndrome
Of the response choices, the most likely diagnosis for the girl in the vignette is trisomy X. Trisomy X or triple X (47,XXX) is a sex chromosome aneuploidy with an incidence of approximately 1 in 1,000 liveborn females. Clinical features are highly variable without clearly defined malformations.
Common reported features include:
Tall stature
Developmental delay: Primarily speech delay; intellectual disability in the mild-to-moderate range (average IQ of 55); learning disabilities in verbal and expressive language
Behavior: Conduct disorder, depression, anxiety, autism spectrum disorder, mood disorder
PREP Pearls
Trisomy X or triple X (47,XXX) is a sex chromosome aneuploidy with highly variable clinical features. The most common features include tall stature, speech delay, and behavioral issues.
Triple X syndrome arises when a nondisjunction event results in an extra chromosome X in the embryo; approximately 60% occur during maternal meiosis I and are associated with advanced maternal age.
A previously healthy, 8-year-old, fully immunized boy is seen in the office for malaise, headache, a 2-day history of fever, and a 10-day history of worsening, nonproductive cough. One week ago he was seen in the emergency department and diagnosed with a viral illness. The boy has no sick contacts, no pets at home, and has not traveled recently.
The boy appears non-toxic. His temperature is 38.2°C, heart rate is 102 beats/min, respiratory rate is 38 breaths/min, and oxygen saturation is 92% in room air. His blood pressure is normal. He has pharyngeal erythema and crackles auscultated in both lung fields. The remainder of his physical examination findings are unremarkable.
Of the following, the MOST likely pathogen causing this boy’s illness is
A. Legionella pneumophila B. Mycoplasma pneumoniae C. Staphylococcus aureus D. Streptococcus pneumoniae
The clinical presentation of the well-appearing boy in the vignette with a gradual onset of illness characterized by prolonged cough, low-grade fever, malaise, pharyngitis, and bilateral crackles on lung examination is suggestive of atypical pneumonia caused by Mycoplasma pneumoniae.
It can be difficult to reliably distinguish bacterial etiologies of community-acquired pneumonia (CAP) from viral etiologies based on clinical findings alone. However, CAP caused by Staphylococcus aureus and Streptococcus pneumoniae is associated with high fever, chills, marked tachypnea, and focal auscultatory findings, which is not consistent with the clinical findings for the boy in the vignette. Atypical infection due to Legionella pneumophila is rare in children, accounting for ≤1% of all pneumonia cases, making this diagnosis unlikely for the boy in the vignette.
PREP Pearls
Mycoplasma pneumoniae is a frequent cause of upper respiratory tract infection and community-acquired pneumonia in school-aged children, adolescents, and young adults.
In ambulatory settings, routine administration of antimicrobial therapy for school-aged children for respiratory tract infection due to suspected Mycoplasma is not indicated.
In children with sickle cell disease, trisomy 21, immunodeficiency disorders, and chronic cardiac or pulmonary disease, Mycoplasma pneumoniae can cause severe, complicated pneumonia.
In hospitalized children with moderate to severe Mycoplasma pneumoniae pneumonia or other severe manifestations, antimicrobial therapy with a macrolide (eg, azithromycin, clarithromycin) may be indicated.
A 12-month-old girl is seen for a health supervision visit. She has been thriving, and her parents do not have any concerns. On physical examination, there is a white pupillary reflex in the right eye. The remainder of her physical examination findings are normal.
Of the following, the BEST next step in this child’s management would be referral to a
A. geneticist B. pediatric oncologist C. pediatric ophthalmologist D. radiation oncologist
The child in the vignette has a white pupillary reflex, known as leukocoria (Item C65A). Leukocoria is a concerning finding, suggestive of retinoblastoma (RB; an abnormal growth of retinoblasts), and must be confirmed by an ophthalmologist.
The geneticist, radiation oncologist, and pediatric oncologist play a role in caring for a child with RB. However, the specialist who must evaluate the child first and make the diagnosis of RB is the pediatric ophthalmologist.
PREP Pearls
Leukocoria is the presenting finding in 50% to 60% of cases of retinoblastoma, and new-onset strabismus may be the presenting finding of retinoblastoma.
A pediatric ophthalmologist must promptly and thoroughly examine a child with leukocoria to evaluate for retinoblastoma.
All children should have screening for a bilateral red reflex at every health supervision visit. The American Academy of Pediatrics guidelines recommend screening from the newborn to 6 months of age, 6 to 12 months of age, 1 to 3 years of age, 4 to 5 years of age, and 6 years and older visits.
A 16-year-old adolescent boy is brought to the emergency department by ambulance after he was involved in a motor vehicle collision. He was the unrestrained driver of a vehicle that hit a tree traveling approximately 40 mph. He complains of nasal and right-sided facial pain. He did not lose consciousness during the accident and was able to self-extricate from the vehicle. He denies neck or back pain. Physical examination reveals swelling and ecchymosis of his right cheek and nasal bridge, and a small amount of blood in his nares. His pupils are equal and reactive to light, and his extraocular movements are intact bilaterally. His dentition is intact, and there is no bleeding from his mouth. Head computed tomography scan does not demonstrate any skull fractures or intracranial hemorrhage. Computed tomography scan of his face shows displaced, comminuted fractures of his right zygomatic arch and ethmoid bones.
Of the following, the BEST next step in this boy’s management is
A. intravenous ampicillin/sulbactam B. magnetic resonance imaging with contrast C. oral analgesia, cold compresses, and nasal decongestant D. urgent evaluation by an otolaryngologist
The boy in the vignette has displaced, comminuted facial fractures and requires urgent evaluation by a pediatric otolaryngologist.
PREP Pearls
Comminuted or displaced facial fractures require urgent evaluation by an otolaryngologist.
Maxillofacial computed tomography (facial bones including orbits, sinuses, mandible, teeth) is the gold standard imaging modality for facial trauma.
For isolated, non-displaced, or minimally displaced facial fractures, conservative management with cold compresses, oral analgesia, and nasal decongestants is appropriate.
A 2-day-old, 38 weeks’ gestation male neonate is seen in the emergency department. This morning, while changing his diaper, his mother noticed blood mixed in the stool. The infant has been fussy for a few hours. He was delivered vaginally during a planned home delivery attended by a neighbor who has experience in performing home births but is not a qualified midwife. The neonate did not receive any medications at the time of or after delivery. He is exclusively breastfed and has been feeding well. The infant’s heart rate is 156 beats/min and blood pressure is 88/35 mm Hg (mean = 43 mm Hg). The infant is awake and fussy. He appears pink and well perfused, with scattered petechiae over the abdomen and blood oozing from the umbilical cord. His abdomen is soft and nontender. Laboratory tests are ordered, and a peripheral intravenous line is placed.
Of the following, the BEST next step in this neonate’s management is to
A. administer ampicillin and gentamicin, intravenously B. administer vitamin K, intravenously C. infuse fresh frozen plasma D. infuse platelets
The neonate in the vignette most likely has hemorrhagic disease of the newborn, and it is evident from the perinatal history that he did not receive 1 mg intramuscular vitamin K after delivery. To prevent further bleeding, it is important to give this patient vitamin K as soon as possible. If no intravenous line is available, intramuscular injection is an appropriate alternative. Coagulation factor replacement and blood product administration, if needed, can occur later.
PREP Pearls
Hemorrhagic disease of the newborn due to vitamin K deficiency can be easily prevented with intramuscular injection of 1 mg of vitamin K after birth.
Vitamin K deficiency should be strongly considered for a newborn presenting with any form of bleeding, especially those exclusively breastfeeding, those delivered at home, and those who did not receive vitamin K prophylaxis after birth.
A 13-year-old adolescent boy with newly diagnosed type 1 diabetes is seen for follow-up. He is doing well with his insulin regimen of multiple daily subcutaneous injections. He takes 12 units of long-acting basal insulin analog once daily at bedtime and boluses of a rapid-acting insulin analog before meals, dosed based on carbohydrate counts (1 unit for every 15 g of carbohydrate) and blood glucose measurements (1 unit for every 50 mg/dL over 150 mg/dL). The boy’s physical examination findings are unremarkable.
His blood glucose log is shown (glucose levels are in mg/dL):
Breakfast
Lunch
Dinner
Bedtime
Day 1
253
118
142
117
Day 2
322
165
133
154
Day 3
196
122
102
125
Day 4
217
131
95
111
Day 5
284
110
118
132
Of the following, the BEST next step in this boy’s management is to
A. ask the boy to check his blood glucose at 3 AM B. ask the boy to check for urine ketones each morning C. increase the basal insulin dose to 13 units D. split the basal insulin dose into 6 units twice daily
The boy in the vignette has newly diagnosed type 1 diabetes, and his blood glucose log demonstrates that he is experiencing elevated blood glucoses in the morning. This pattern is most commonly due to too little (dawn effect) or too much (Somogyi effect) bedtime basal insulin. A blood glucose level checked in the middle of the night (usually between 2-3 AM) can help distinguish between these two effects. An elevated overnight glucose level indicates too little basal insulin, and a low level indicates too much basal insulin.
PREP Pearls
In type 1 diabetes, elevated morning blood glucose levels may be due to either too little (dawn effect) or too much basal insulin (Somogyi effect).
Checking a blood glucose level in the middle of the night (2-3 AM) can help distinguish between the dawn and Somogyi effects.
Understanding the pharmacodynamics of insulin analogs is essential for making effective insulin dose adjustments
A 6-year-old, previously healthy boy is brought to the urgent care center with a 2-week history of cough and low-grade fever. On physical examination, his temperature is 38°C, his oxygen saturation is 94% in room air, and he has crackles over his left lung base. Chest radiography shows a left lower lobe consolidation and a radiolucent lesion with a thin cortical rim with a fallen leaf fragment in the metaphyseal region of the right proximal humerus (Item Q70). The boy has no history of arm pain. Right upper extremity physical examination findings are normal.
Of the following, the BEST next step in management of this boy’s incidental finding is
A. bone biopsy B. bone scan C. magnetic resonance imaging D. observation
The boy in the vignette has an incidental radiographic finding of a simple bone cyst, also known as a solitary bone cyst or unicameral bone cyst (UBC). This is a benign pediatric bone tumor representing 3% of primary bone tumor biopsies. Unicameral bone cysts occur most often between ages 4 and 10 years; the etiology remains unknown. Most UBCs are located in the metaphysis of the proximal humerus or femur but can occur in other appendicular or axial bones. They are generally painless and discovered incidentally but can be mildly painful, reflecting a microscopic fracture. Trauma to the area may result in an abrupt pathologic fracture presenting with pain, edema, erythema, warmth, decreased range of motion, and deformity.
PREP Pearls
The diagnosis of a unicameral bone cyst can be made using radiography; typical features include location in the metaphysis that extends to, but does not cross, the physis; radiolucency with a thin cortical rim; and the pathognomonic sign of the “fallen leaf” fragment representing a portion of fractured cortical bone settling to the dependent part of the fluid-filled cyst.
The differential diagnosis of a unicameral bone cyst includes an aneurysmal bone cyst, fibrous dysplasia, and enchondroma.
Observation is reasonable for asymptomatic and small unicameral bone cysts. Indications for surgical intervention include persistent pain, recurrent fracture, and prevention of deformity.
A 14-year-old adolescent girl is seen in the office for follow up of dysmenorrhea. Based on the results of laboratory evaluation performed at her initial visit for this condition 9 months ago, she was diagnosed with iron-deficiency anemia and type 2 diabetes (hemoglobin A1C, 6.6%). Today, she reports ongoing heavy and painful menstrual cycles as well as increasing fatigue; she is otherwise well. She has never had sexual intercourse. The girl’s vital signs are normal. Her body mass index has increased over the past 3 months from 98% to 99% for age and sex. Her current prescribed medications are ferrous sulfate 325 mg every other day, naproxen sodium 440 mg twice daily at the time of her menses, and metformin 1,000 mg twice daily.
Laboratory data are shown:
Laboratory Test
Result
Hemoglobin
9.3 g/dL (93 g/L)
Mean corpuscular volume
78 fL
Mean corpuscular hemoglobin concentration
34.2 g/dL (342 g/L)
Ferritin
2 ng/mL (2 µg/L)
Hemoglobin A1C
6.7%
Of the following, the BEST next step in this girl’s management is
A. assess adherence by asking open-ended questions B. decrease the dose of metformin for 2 weeks C. increase her dose of ferrous sulfate to 325 mg daily D. refer her to a hematologist for possible iron infusion therapy
The adolescent in the vignette has recently been diagnosed with chronic medical conditions that require daily medications for long-term management. Medication adherence is difficult for adolescents, especially if parents are not monitoring medications closely to increase accountability. The best next step in the management of the girl in the vignette is to assess medication adherence by asking her open-ended questions. For example, “Are you in charge of your own medications?” “Do you ever accidentally forget to take them?” “Do any of them cause you problems?” The physician should not increase or decrease the dosage of her medications until it is assessed if the patient is taking their medications correctly. While referral to hematology for iron infusion therapy may be needed at some point, this step should not occur before open-ended inquiry regarding her medication adherence.
PREP Pearls
Adherence to medications and medical treatment plans can be challenging for adolescents due to various factors including peer factors, striving for “normalcy,” forgetting the medication, physical well-being (eg, side effect concerns), and parent conflict.
Physicians can assess medication adherence with open-ended, nonjudgmental questions.
Physicians should participate as team members with parents and adolescents to problem solve how to improve adherence.
A 16-month-old boy is seen for a routine health supervision visit. He has been well since his last visit 4 months ago and has not had any recent illness. The boy was breastfed until 12 months of age, when he transitioned to cow milk. He started eating solid foods at 6 months of age, and occasionally eats crackers, but mainly consumes milk (about 48 oz/day). He also drinks juice. The boy’s activity level, stools, and urine output are normal. His heart rate is 100 beats/min. He looks well nourished. His lips, tongue, and conjunctiva are pale. There is no scleral icterus, and his mucous membranes are moist. He has a 2/6 systolic ejection murmur at the left sternal border. The remainder of his physical examination findings are normal.
Laboratory data are shown:
Laboratory Test
Result
White blood cell count
6,800/µL (6.8 × 109/L)
Hemoglobin
5.5 g/dL (55 g/L)
Platelet count
480 × 103/µL (480 × 109/L)
Mean corpuscular volume
65 fL
Red cell distribution width
18%
Neutrophils
41.2%
Lymphocytes
54.3%
Monocytes
4.2%
Eosinophils
0.3%
Reticulocyte
1%
Absolute reticulocyte count
44 × 103/µL (44 × 109/L)
Iron
20 µg/dL (3.6 µmol/L) (reference range, 45-165 µg/dL [8.1-30 µmol/L])
Total iron-binding capacity
530 µg/dL (95 µmol/L) (reference range, 220-430 µg/dL [39.4-77 µmol/L)
Unsaturated iron-binding capacity
507 µg/dL (90.8 µmol/L) (reference range, 110-370 µg/dL [19.7-66.2 µmol/L])
Iron saturation
3.7% (reference range, 14-50%)
Ferritin
<5 ng/mL (<5 µg/L) (reference range, 30-400 ng/mL [30-400 µg/L])
Of the following, the BEST next best step in this boy’s management is to
A. give a 20-mL/kg normal saline bolus, intravenously B. give a 20-mL/kg transfusion of packed red blood cells C. give a dose of iron, intravenously D. recommend dietary changes and close follow-up
The boy in the vignette has severe iron-deficiency anemia (hemoglobin level of 5.5 g/dL [55 g/L]) due to inadequate dietary intake of iron. The best next management step is to give a dose of iron intravenously. Parenteral iron is indicated for children who require a rapid and/or short treatment course, such as the boy in the vignette, and for children who do not tolerate oral iron supplementation or have ongoing blood loss or gastrointestinal disease.
The boy in the vignette is hemodynamically stable so he does not require an immediate increase in his hemoglobin level. A transfusion of packed red blood cells would be indicated to rapidly increase the hemoglobin level if the boy had symptoms such as severe fatigue, syncope or near syncope, or significant tachycardia. An intravenous normal saline bolus would be indicated if the boy was hemodynamically unstable. Normal saline boluses should be given with caution in the setting of anemia because the dilutional effect causes worsening of the anemia and decreased oxygen-carrying capacity, and could lead to heart failure. Although dietary changes should be recommended for the boy, his severe anemia necessitates more rapid iron replacement.
Oral iron supplementation would also be appropriate for the boy in the vignette. The ferrous form of iron, at a minimum dose of 3 mg/kg/day, is sufficient to replenish iron stores over several months (3 or more). Very high doses of oral iron (eg, 6 mg/kg/day) can cause bowel irritation and constipation, which may lead to decreased adherence.
PREP Pearls
Parenteral iron is indicated for children with iron deficiency who are hemodynamically stable and require a rapid and/or short treatment course, do not tolerate oral iron supplementation, or have ongoing blood loss or gastrointestinal disease.
A packed red blood cell transfusion is indicated for the treatment of symptomatic anemia (eg, severe fatigue, syncope or near syncope, or significant tachycardia).
An increase in the reticulocyte percentage is an early indicator of recovery during the treatment of iron-deficiency anemia.
A 4-year-old boy is seen in the office with fever, rash, and decreased oral intake. He has had fevers for 5 days, and a new skin rash was noted this morning. He has had no nasal congestion, cough, vomiting, or diarrhea. His temperature is 38.5°C, heart rate is 130 beats/min, respiratory rate is 30 breaths/min, blood pressure is 85/55 mm Hg, and oxygen saturation is 99% in room air. He is irritable but consolable. His conjunctivae are injected without drainage bilaterally, lips and tongue are erythematous without blisters or exudate, and he has cervical lymphadenopathy on the right. A diffuse maculopapular erythematous rash is noted on his trunk and extremities as well as erythema of his hands and feet. The remainder of his physical examination findings are unremarkable.
Of the following, the BEST next management step for this boy is to
A. administer acetylsalicylic acid orally at 10 mg/kg per day B. administer intravenous immunoglobulin at 2 g/kg C. obtain a complete echocardiogram D. obtain laboratory data including C-reactive protein level
The boy in the vignette has a history and physical examination findings that meet criteria for classic Kawasaki disease (KD). The best next step for him is treatment with 2 mg/kg of intravenous immunoglobulin to reduce the risk of coronary artery aneurysm formation. Although children with KD are treated with acetylsalicylic acid at the time of diagnosis, a dose of 10 mg/kg per day is not adequate. Echocardiography and laboratory evaluation are important to the complete evaluation of KD, but they are not as urgent as the administration of immunoglobulin.
PREP Pearls
Classic Kawasaki disease is diagnosed when a child has fever for 5 or more days and at least 4 of the following 5 clinical findings:
Erythema and cracking of the lips, strawberry tongue, and/or erythema of oral and pharyngeal mucosa
Bilateral bulbar conjunctival injection without exudate
Maculopapular diffuse erythematous or erythema multiforme-like rash
Erythema and edema of the hands and feet in acute phase and/or periungual desquamation in subacute phase
Cervical lymphadenopathy (≥1.5-cm diameter), usually unilateral
Children with a prolonged fever with fewer than 4 of the classic features may have atypical or incomplete Kawasaki disease; this diagnosis may be supported by laboratory and echocardiographic findings.
Treatment of children with Kawasaki disease with 2 mg/kg of intravenous immunoglobulin is required to reduce the incidence of coronary artery aneurysms; acetylsalicylic acid is used to treat inflammation and block platelet function.
A 14-year-old adolescent boy is admitted to the hospital for management of anemia, hematochezia, and hypovolemia. He has a 6-month history of recurrent hematochezia and diffuse abdominal pain with worsening symptoms over the past 3 weeks, now with 5 to 6 episodes of bloody diarrhea daily. He reports bilateral knee pain, a 5-lb weight loss, fevers, and mouth sores over the past 2 weeks. Physical examination reveals a pale and tired-appearing adolescent. His heart rate is 120 beats/min, blood pressure is 130/70 mm Hg, weight is 57 kg (60th percentile for age), height is 160 cm (18th percentile for age), and body mass index is 25 kg/m2 (90th percentile for age). He has a soft systolic murmur, a diffusely tender abdomen without rebound or guarding, and his rectal examination findings are notable for grossly bloody stools without other abnormality.
After the administration of intravenous fluids and packed red blood cells, his heart rate normalizes. Endoscopy with colonoscopy is performed after a bowel cleanout. The upper endoscopy findings are normal. The colonoscopy demonstrates severe mucosal erythema, friability and edema, with exudate from the rectum to the cecum (Item Q74); pathology demonstrates severe chronic colitis without granulomas with a normal terminal ileum. Magnetic resonance enterography reveals a thickened colonic bowel wall with a normal-appearing small intestine.
Of the following, the BEST treatment for the adolescent’s condition is
A. exclusive enteral nutrition B. intravenous corticosteroids C. oral azathioprine D. oral tacrolimus
The adolescent in the vignette has findings consistent with ulcerative colitis (UC); the constellation of signs, symptoms, and laboratory findings meets the criteria for acute severe colitis (ASC). The initial therapy recommended for ASC includes intravenous corticosteroids and supportive care (including treatment of anemia and nutritional support as clinically warranted).
A 3-year-old girl (Item Q113) is seen for a health supervision visit with her adoptive parents. Prenatal history is not known. Gross motor skills are mildly delayed. Her other developmental milestones are appropriate for age. Her weight and height are at the 3rd percentile, and her head circumference is at the 5th percentile.
Of the following, the girl’s clinical findings are MOST likely due to
A. congenital cytomegalovirus infection
B. folate deficiency during pregnancy
C. maternal alcohol consumption during pregnancy
D. uncontrolled maternal diabetes during pregnancy
The girl in the vignette demonstrates features of fetal alcohol spectrum disorder (FASD) including growth restriction, gross motor delay, and the physical findings shown in Item C113A (midface hypoplasia, a relatively smooth philtrum, small palpebral fissures, and a narrow upper lip with a poorly defined “cupid’s bow”). Fetal alcohol spectrum disorder results from maternal alcohol consumption during pregnancy. There is no safe limit for alcohol consumption during pregnancy; FASD can occur with any level of alcohol intake. Both the American Academy of Pediatrics and the American College of Obstetricians and Gynecologists recommend complete abstinence from alcohol for preconceptional and pregnant women. The clinical features of FASD include:
Growth restriction (height and/or weight ≤10%)
Dysmorphic features (small palpebral fissures, thin vermillion border, flat nasal bridge, smooth philtrum, railroad track appearance to the ear, hockey stick palmar crease, and midface hypoplasia)
Central nervous system abnormalities
Congenital cytomegalovirus infection can present with prematurity, intrauterine growth retardation, microcephaly, poor feeding, thrombocytopenia, hepatosplenomegaly, intracranial calcifications, sensorineural hearing loss, and chorioretinitis. While developmental delay may be seen, there are no dysmorphic facial features associated with the condition.
A 5-year-old boy who recently emigrated from a refugee camp in Burma is brought to the office with complaints of abdominal pain, anorexia, and occasional watery diarrhea for more than 3 weeks. He is afebrile with normal vital signs. He has abdominal distention but no hepatomegaly or splenomegaly. The remainder of his physical examination findings are normal.
Laboratory data are shown:
Laboratory Test
Result
White blood cell count
12,000/μL (12.0 × 109/L)
Neutrophils
68%
Lymphocytes
26%
Monocytes
2%
Eosinophils
4%
Hemoglobin
11.7 g/dL (117 g/L)
Platelet count
274 × 103/μL (274 × 109/L)
A stool specimen is obtained for microscopic examination (Item Q114).
Of the following, the MOST likely pathogen causing this boy’s illness is
A. Ancylostoma duodenale B. Ascaris lumbricoides C. Necator americanus D. Trichuris trichura
The boy in the vignette is infected with Ascaris lumbricoides, an intestinal roundworm commonly found in locations with tropical climates (eg, Burma). The boy’s symptoms (abdominal pain, anorexia, watery diarrhea), physical examination finding (abdominal distention), and characteristic appearance of the fertilized eggs on microscopic examination of a stool specimen (oval shape surrounded by a thick shell with an external proteinaceous layer) are consistent with A lumbricoides infection.
A previously healthy, 7-year-old girl is seen in the office for right knee pain and swelling that developed and has been worsening over the past several days. She also has pain while walking. She had an episode of acute pharyngitis 3 weeks ago that resolved without intervention. On physical examination, the girl has normal vital signs for her age. She has erythema and swelling of the right knee with pain on motion. On chest auscultation, there is a long, high-pitched systolic murmur at the apex. There is an erythematous, serpiginous rash throughout her trunk that blanches. The remainder of her examination findings are unremarkable.
Of the following, the BEST test to establish the child’s diagnosis is
A. antistreptolysin O antibody titer B. C-reactive protein C. echocardiography D. rheumatoid factor
The child in the vignette meets several of the modified Jones criteria for diagnosing acute rheumatic fever (ARF): arthritis, erythema marginatum, and carditis. To definitively make the diagnosis, however, one needs to also demonstrate evidence of a prior group A streptococcal (GAS) infection. The best test to establish the diagnosis of ARF is antistreptolysin O titer. A C-reactive protein level and echocardiography may be useful in the complete assessment and management of the condition but will not clarify the underlying diagnosis. In general, a rheumatoid factor level is unlikely to be helpful in the diagnosis of knee pain and swelling in a young child and is not recommended to make the diagnosis in this child with a clinical picture suggestive of ARF.
A 2-year-old, previously healthy boy is brought to the emergency department for difficulty walking. Two weeks ago he developed an upper respiratory infection that self-resolved. Over the past 4 days, he has had difficulty standing and walking, and requires support to take steps such as holding onto furniture or his parents’ hands. When he sits, he nods his head and has trouble feeding himself. His parents deny recent trauma or ingestion.
On physical examination, the boy is alert and playful in his mother’s lap, resting against her. He orients when his name is called. His pupils are equal, round, and reactive to light, extraocular movements are intact with a few beats of end gaze nystagmus bilaterally, facies are symmetric, and tongue is midline. Muscles have normal bulk, tone, and strength with symmetric, full movements. Deep tendon reflexes are initially difficult to elicit but are 2 throughout; toes are downgoing to plantar stimulation. Sensation is intact. When reaching for objects he has mild dysmetria bilaterally. He maintains a seated posture without support but has titubation. He can walk with significant support (holding his mother’s hands) with a wide-based gait.
Results of a complete blood cell count, comprehensive metabolic panel, toxicology screen (urine and serum), and brain magnetic resonance imaging are normal.
Of the following, the MOST likely diagnosis for this boy is
A. acute cerebellar ataxia B. acute disseminated encephalomyelitis C. developmental coordination disorder D. Miller-Fisher syndrome
The boy in the vignette has acute cerebellar ataxia, a self-limited, postinfectious disorder primarily affecting children younger than 5 years. Affected children exhibit truncal and gait ataxia, often coming to medical attention due to the inability to stand or sit without support. Dysmetria and titubation (head nodding) while sitting are further evidence of cerebellar dysfunction. Affected children are otherwise well appearing with normal mental status. Symptoms are usually maximal at onset and gradually improve over days to weeks, rarely months. Management is supportive. There is no evidence that immune therapy (eg, corticosteroids or intravenous immunoglobulin [IVIG]) affects outcome.
PREP Pearls
The differential diagnosis of balance disturbances in childhood is broad and can localize anywhere within the nervous system. A careful history and physical examination are critical for appropriate localization, evaluation, and diagnosis.
Ataxia refers to an absence of coordination and can be acute, subacute, or chronic depending on the time course of symptoms.
Acute cerebellar ataxia is a self-limited, post-infectious disorder presenting with acute onset of truncal and gait ataxia, often in young children. Treatment is supportive.
A 13-year-old adolescent girl is seen in the office for concern about absent menses. She has been healthy except for multiple ear infections as an infant. The girl is doing well in school and participates in club soccer. Her sexual maturity rating is 1 for breast development and 3 for pubic hair. The remainder of the girl’s physical examination findings are unremarkable. Her growth charts are shown in Item Q119A and Item Q119B.
Of the following, the test/assessment MOST likely to reveal this girl’s diagnosis is a
A. bone age radiograph B. calorie intake and expenditure assessment C. karyotype D. tissue transglutaminase IgA antibody level
The girl in the vignette has Turner syndrome. The best test to reveal her diagnosis is a karyotype, which would show an absent or missing part of a sex chromosome (45,X or a variant). Her short stature, absent breast development, and history of multiple ear infections as an infant are all features of Turner syndrome.
Delayed puberty in girls is defined as lack of breast development by age 13 years. In adolescents with delayed puberty, measurement of gonadotropins (luteinizing hormone and follicle-stimulating hormone [FSH]) is indicated to determine whether the pubertal failure is primary (ovarian/testicular) or secondary (hypothalamic-pituitary). In adolescent girls with Turner syndrome, the FSH is often elevated due to primary ovarian failure. The girl in the vignette has a sexual maturing rating of 3 for pubic hair, which is secondary to adrenal androgen production.
Bone age radiography is an important part of the evaluation of short stature and delayed puberty. Typically, conditions that delay puberty also delay bone age as estradiol is the primary hormone responsible for growth plate maturation. Although bone age radiography may help in treatment planning for the girl in the vignette, it is not specific enough to determine the etiology of her delayed puberty. A delayed bone age in an otherwise healthy child or adolescent with a normal growth rate aids in the diagnosis of constitutional delay of growth and puberty.
A 2-day-old neonate is transferred to the neonatal intensive care unit for a dusky appearance. On physical examination, he has a heart rate of 160 beats/min, respiratory rate of 60 breaths/min, blood pressure of 70/55 mm Hg in the right arm, and right upper and lower extremity oxygen saturations of 78% in room air. He is tachypneic but comfortable, without increased work of breathing. Breath sounds are clear. He has a normal S1, a single S2, and a continuous murmur heard in the left side of the chest; the liver edge is palpable 2 cm below the right costal margin; and his extremities are warm and well perfused. A chest radiograph is obtained (Item Q120).
Of the following, the BEST next step in this neonate’s management is to
A. initiate epinephrine infusion B. initiate inhaled nitric oxide C. initiate prostaglandin infusion D. intubate for respiratory failure
he neonate in the vignette is cyanotic, with a room air oxygen saturation of 78% measured in the right upper and lower extremities. He is comfortably tachypneic with good perfusion. On auscultation, he has clear breath sounds, a single S2, and a continuous murmur. The chest radiograph (Item C120A) shows oligemic lungs without infiltrate, a boot-shaped heart, and a right aortic arch. These findings are consistent with tetralogy of Fallot with pulmonary atresia. Although this diagnosis will need to be confirmed by echocardiography, for this ductal-dependent condition it is important to first initiate prostaglandin infusion to prevent further closure of the patent ductus arteriosus (PDA). The neonate does not have evidence of low cardiac output, so administration of epinephrine is not indicated. He does not have evidence of respiratory failure, so intubation is not indicated. Inhaled nitric oxide would not correct his hypoxemia.
The mnemonic 5 Ts and H is helpful for remembering many of the types of cyanotic congenital heart disease:
Tetralogy of Fallot
Total anomalous pulmonary venous connection
Transposition of the great arteries
Tricuspid atresia
Truncus arteriosus
Hypoplastic left heart syndrome
A 15-year-old adolescent girl is brought to the emergency department for a 48-hour history of blurry vision and pain with movement of the left eye. Today, she is unable to see anything out of her left eye. She complains of mild, left frontal headache but denies fever or eye discharge. There is no history of recent illness or eye injury. Her vital signs are normal, and she is in no acute distress. Her cornea and conjunctivae are clear without discharge or injection. She has mild pain with movement of her left eye but has full range of extraocular movement. When light is directed at her right pupil, there is normal constriction of both pupils. When light is directed at her left pupil, both pupils remain dilated. Fundoscopy shows blurred optic disc margins on the left with sharp margins on the right. The remainder of her physical examination findings are normal.
Of the following, the MOST likely diagnosis for this adolescent is
A. migraine variant B. optic neuritis C. orbital cellulitis D. pseudotumor cerebri
The adolescent in the vignette has findings consistent with left optic neuritis. The swinging flashlight test, also known as the Marcus Gunn pupil test, is performed to detect an afferent pupillary defect. The test is performed by shining a light on one eye for 2 seconds and then alternating the light from eye to eye. Normally, both pupils constrict in response to shining the light on one pupil. If the pupils remain dilated, an afferent pupillary defect is likely, indicating disease of the retina or optic nerve. The adolescent has a lack of pupillary response on the left side with a normal direct and consensual response on the right side, indicating a left afferent pupillary defect. This finding, along with blurring of her left optic disc, makes optic neuritis the most likely diagnosis.
The first-line treatment of pediatric optic neuritis is high-dose, pulsed methylprednisolone for 3 to 5 days followed by a taper over 2 weeks. Children and adolescents resistant to steroid therapy may benefit from treatment with intravenous immunoglobulin and plasmapheresis.
PREP Pearls
Optic neuritis can present with acute vision loss and findings of an afferent pupillary defect with blurring of the optic disc margins.
Evaluation of children and adolescents with suspected optic neuritis should include magnetic resonance imaging of the brain and orbits.
Optic neuritis can be the initial clinical presentation of demyelinating disorders such as multiple sclerosis.
A 3-year-old girl is brought to the emergency department (ED) for difficulty breathing. She developed a sudden, severe sore throat and fever today. Her vital signs include a temperature of 40.1°C, heart rate of 176 beats/min, respiratory rate of 26 breaths/min, blood pressure of 76/59 mm Hg, and oxygen saturation of 91% in room air. On arrival at the ED, the girl appears toxic and has severe respiratory distress and difficulty swallowing. She is sitting upright and holding her head in an extended position while resting her arms on her knees. She has stridorous respirations associated with tachypnea and suprasternal retractions.
Of the following, the BEST next step in this girl’s management is
A. administration of intravenous ceftriaxone B. administration of intravenous corticosteroid C. administration of nebulized racemic epinephrine D. rapid sequence induction and intubation
The girl in the vignette is displaying signs and symptoms of airway obstruction concerning for epiglottitis. The best next step in her management is rapid sequence induction and intubation. Other interventions, such as nebulizer treatments, intravenous corticosteroids or antibiotics, or imaging studies should not be undertaken until a definitive airway is secured. Every effort should be made to keep the child as calm and quiet as possible until the airway is secured.
PREP Pearls
Epiglottitis is a rare life-threatening condition that results in severe swelling of the epiglottis, which can lead to complete airway obstruction.
The clinical presentation of epiglottis is an overtly toxic-appearing child with sudden onset and rapid progression of fever, drooling, stridor, muffled voice, and difficulty swallowing. The classic diagnostic triad, the “3 Ds,” includes drooling, dysphagia, and distress.
The critically important first step in the management of a child with suspected epiglottitis is obtaining a definitive and secure airway with an endotracheal tube
A 14-year-old adolescent boy is seen for a preparticipation physical examination to play soccer. He is healthy with no concerns voiced by him or his parents. His physical examination findings are normal except for a systolic ejection click heard best at the right upper sternal border. Further inquiry reveals that the boy’s paternal grandfather had aortic valve surgery at age 55 years. The boy is referred to a pediatric cardiologist.
Of the following, the boy’s MOST likely diagnosis is
A. bicuspid aortic valve B. partial anomalous pulmonary venous return C. pulmonary valve stenosis D. ventricular septal defect
The boy in the vignette has a systolic ejection click heard at the right upper sternal border, suggestive of aortic valve pathology, and a family history (paternal grandfather) of aortic valve pathology. Of the response choices, bicuspid aortic valve (BAV) is the most likely diagnosis. He should be evaluated by a pediatric cardiologist; echocardiography is needed to confirm the diagnosis. If there is no significant valve dysfunction or aortic dilation, children with BAV can often play sports and exercise without restriction.
PREP Pearls
Bicuspid aortic valve, in which the aortic valve develops with only 2 leaflets instead of the normal 3, is one of the most common congenital heart defects.
Bicuspid aortic valve occurs in males more than females and can be familial.
Individuals with bicuspid aortic valve can develop valve dysfunction (stenosis, regurgitation, or both), aortic dilation, or aortic aneurysm.
A 7-year-old girl is evaluated for a 1-week history of a dry, hacking cough. Today, she had an episode of posttussive emesis. The girl appears well. She has a temperature of 37.5°C, heart rate of 90 beats/min, respiratory rate of 18 breaths/min, and oxygen saturation of 100% in room air. Her lungs are clear on auscultation. Bilateral subconjunctival hemorrhages and a few scattered petechiae are noted over the neck. Her parents are concerned that the girl’s siblings, ages 6 weeks and 5 years, will catch her illness. The children are all up to date with their immunizations.
Of the following, over the next 21 days, the MOST appropriate postexposure management is to
A. monitor only the 5-year-old sibling for symptoms B. monitor only the 6-week-old sibling for symptoms C. start azithromycin treatment for both siblings D. start trimethoprim-sulfamethoxazole treatment for both siblings
Pertussis postexposure prophylaxis with azithromycin is recommended for all close contacts of the index case irrespective of immunization status. Individuals are considered close contacts if they are either household members or asymptomatic persons who had face-to-face exposure within 3 feet of a symptomatic individual in a childcare facility, school, or health care setting. Exposure is defined as direct contact with respiratory, oral, or nasal secretions or sharing the same space with the symptomatic individual for 1 hour or more. Prompt postexposure prophylaxis is recommended, given the high transmission risk (secondary attack rate of 80% in susceptible persons) and effectiveness of azithromycin. Trimethoprim-sulfamethoxazole is reserved for prophylaxis in children who cannot tolerate azithromycin; it is contraindicated in infants younger than 2 months. All exposed individuals should be monitored for symptoms for 21 days.
PREP Pearls
Pertussis is a highly contagious infection with a typical duration of 6 to 10 weeks.
There are 3 stages of pertussis infection: catarrhal (upper respiratory infection symptoms), lasting 1 to 2 weeks; paroxysmal (repeated coughing spells), lasting 2 to 4 weeks; and convalescent (improving cough), lasting 4 to 12 weeks.
Household members and close contacts of individuals diagnosed with pertussis infection should receive postexposure prophylaxis with azithromycin, irrespective of their immunization status.
A 4-day-old male neonate is seen in the office for a health supervision visit. The neonate was born at term, had an uneventful prenatal and nursery course, and was discharged 2 days after birth. His mother is planning to exclusively breastfeed and asks if any nutrient supplements are recommended for her son.
Of the following, the BEST recommendation is
A. start vitamin D and iron supplementation at age 4 months B. start vitamin D and iron supplementation now C. start vitamin D supplementation now and iron supplementation at age 4 months D. vitamin D and iron supplementation are not necessary
The breastfed neonate in the vignette should start vitamin D supplementation now and iron at age 4 months. Although breast milk provides excellent nutrition during the first year after birth, the vitamin D and iron content in breast milk is not adequate to meet the infant’s nutritional needs.
A 1-day-old newborn was delivered at 39 weeks’ gestation via normal spontaneous vaginal delivery to a 21-year-old primigravida mother. The pregnancy was complicated by oligohydramnios. Prenatal ultrasonography revealed bilateral hydronephrosis. The neonate has had 1 small void since birth. His vital signs are normal for age, and his physical examination findings are unremarkable.
Postnatal renal ultrasonography reveals severe bilateral hydroureteronephrosis with a thick-walled urinary bladder.
Of the following, the BEST next step in this neonate’s evaluation is to order
A. a diuretic renal scan B. magnetic resonance imaging of the kidneys and bladder C. repeat renal ultrasonography after 1 week D. voiding cystourethrography
The male neonate in the vignette most likely has posterior urethral valves (PUVs). This diagnosis is supported by the history of bilateral antenatal hydronephrosis, maternal oligohydramnios, and severe bilateral hydroureteronephrosis with a thick-walled urinary bladder on postnatal renal ultrasonography. The best next step in his evaluation is to perform voiding cystourethrography (VCUG). The VCUG would likely demonstrate the classic findings of a dilated posterior urethra during the voiding phase, trabeculated bladder, and perhaps vesicoureteral reflux (VUR) (Item C126).
PREP Pearls
Posterior urethral valves are abnormal mucosal folds within the lumen of the posterior urethra that obstruct the flow of urine.
Children with suspected posterior urethral valves should have voiding cystourethrography (VCUG) performed. A dilated posterior urethra on VCUG is consistent with the diagnosis.
Children with posterior urethral valves are at increased risk of chronic kidney disease, even with prenatal diagnosis and early intervention.
A 2-year-old girl is seen for hives of 24 hours’ duration. The girl woke up yesterday with scattered lesions on her torso. Since onset, the lesions have come and gone in various areas of her torso. Two weeks ago, she was diagnosed with a viral upper respiratory tract infection and acute otitis media for which she was prescribed amoxicillin. She finished the amoxicillin 4 days ago. For the past 2 days, the girl has had a temperature of 38°C, mild cough, and rhinorrhea. She has continued at her usual activity level and is eating a wide variety of foods. Her medical history is unremarkable. Her mother has hypothyroidism and her older brother has a peanut allergy. The girl’s vital signs are normal. Physical examination findings are remarkable only for clear rhinorrhea and scattered 1- to 5-cm diameter hives on her torso.
Of the following the MOST likely cause of this girl’s hives is
A. amoxicillin B. food allergy C. hypothyroidism D. viral illness
The most likely cause for the hives exhibited by the girl in the vignette is a viral illness. Viral infections cause almost 60% of acute urticaria seen in children, and this girl has clear symptoms of a viral illness. Amoxicillin is a less likely cause of this girl’s urticaria because urticaria caused by amoxicillin usually does not have a delayed onset. Urticaria from food allergies usually presents within 2 hours of exposure. Hypothyroidism, specifically autoimmune hypothyroidism, may predispose some children to urticaria, but in these cases, the urticaria is usually chronic, lasting more than 6 weeks.
PREP Pearls
The most common cause of acute urticaria in children is a viral infection.
The first-line treatment for both acute and chronic urticaria is a nonsedating antihistamine such as cetirizine or loratadine.
Children with cold-induced urticaria should be prescribed an epinephrine auto-injector because of the risk of anaphylaxis.
A 14-year-old adolescent boy is brought to the emergency department by ambulance after his parents found him unresponsive in his bedroom. Upon arrival at the home, the emergency medical services team witnessed a 5-minute generalized tonic clonic seizure for which the paramedic administered diazepam 2 mg intravenously. The boy has a history of depression and attention-deficit/hyperactivity disorder. His current medications include melatonin, amitriptyline, and dexmethylphenidate.
In the emergency department, the boy has a temperature of 38.3°C, heart rate of 188 beats/min, respiratory rate of 10 breaths/min, blood pressure of 72/45 mm Hg, and oxygen saturation of 90% in room air. He appears sleepy, without apparent seizure activity. His physical examination findings are significant for dilated pupils; flushed, dry skin; and dry mucous membranes. He has shallow breathing and decreased respiratory effort. The remainder of his physical examination findings are unremarkable.
The boy is intubated, and his airway is secured. Results of a complete blood count and comprehensive metabolic panel are normal. Shortly after intubation he is noted to have rhythm changes on the cardiac monitor; 12-lead electrocardiography is obtained and shown (Item Q128).
Of the following, the BEST next step in this boy’s management is to administer intravenous
A. flumazenil B. magnesium sulfate C. physostigmine D. sodium bicarbonate
The adolescent in the vignette is demonstrating classic anticholinergic findings (seizure; respiratory depression; dilated pupils; dry, flushed skin; tachycardia; and arrhythmia) caused by an overdose of amitriptyline, a tricyclic antidepressant (TCA). The treatment for TCA-induced, wide complex ventricular tachycardia and resulting hypotension is administration of intravenous sodium bicarbonate, with the goal of raising the serum pH to approximately 7.5.
Flumazenil, the antidote for a benzodiazepine overdose, is contraindicated in individuals with TCA overdose as it may induce seizures. Flumazenil should also not be administered to individuals with an unknown ingestion, as it may precipitate seizures in those who are benzodiazepine dependent.
PREP Pearls
An electrocardiogram should be obtained immediately in a child with suspected or confirmed ingestion of a tricyclic antidepressant.
Sodium bicarbonate is the mainstay of treatment in children with tricyclic antidepressant toxicity and should be initiated in any child who is hemodynamically unstable, acidotic, hypotensive, or exhibiting seizure activity or a QRS interval prolongation of more than 100 milliseconds.
The goal of sodium bicarbonate therapy for tricyclic antidepressant–induced QRS interval prolongation is to narrow the QRS complex and achieve and maintain a serum pH of 7.5.
A 3-year-old girl in the pediatric intensive care unit has acute respiratory failure from respiratory syncytial virus bronchiolitis. She is in marked respiratory distress, with a respiratory rate of 60 breaths/min and oxygen saturation of 73% in room air. She is somnolent and lethargic. Auscultation of the lungs demonstrates diffuse, coarse rales, wheezes, and rhonchi. The decision is made to perform rapid-sequence endotracheal intubation, after which her oxygen saturation is 98% on an FiO2 of 45%. Approximately 20 minutes after initiation of mechanical ventilation, the girl has an acute oxygen desaturation to 75% which does not improve when 100% FiO2 is provided by the ventilator. Visual inspection of her chest reveals equal but restricted chest rise, and auscultation reveals diminished breath sounds in all lung fields.
Of the following, the BEST next step in this girl’s management is
A. chest radiography B. manual ventilation with a resuscitation bag C. needle thoracostomy of the chest D. suctioning of the endotracheal tube
The girl in the vignette has acute respiratory failure requiring intubation and mechanical ventilation. Given her ongoing hypoxia despite the provision of 100% FiO2 by the ventilator, equal but diminished breath sounds, and equal chest rise, the best next step in management is to provide manual ventilation with a resuscitation bag via the endotracheal tube to both assess and manage the cause of ventilation failure.
Finally, suctioning of the endotracheal tube can alleviate mucus or secretion-related endotracheal tube obstruction. However, given the ongoing significant oxygen desaturations, manual ventilation with a resuscitation bag is indicated before suctioning to attempt to restore normal oxygenation quickly.
PREP Pearls
When acute hypoxia occurs in mechanically ventilated children, manual ventilation with a resuscitation bag should be performed with 100% FiO2 until the underlying cause of hypoxia can be addressed and corrected.
Endotracheal intubation may be required for children with respiratory failure, impending respiratory arrest, or those with neurologic impairment or severely altered mental status where airway protective reflexes cannot be maintained.
A common mnemonic used to evaluate and manage a successfully intubated patient who subsequently develops hypoxemia is DOPE: Displacement/Dislodgement of the endotracheal tube, Obstruction of the endotracheal tube, Pneumothorax, and Equipment failure.
A plan is made to treat the adolescent with antibiotics and draw repeat titers for Bartonella henselae in 4 weeks.
Of the following, the BEST next step in management of this adolescent is to administer
A. amoxicillin B. azithromycin C. cephalexin D. penicillin
The adolescent in the vignette has unilateral cervical lymphadenopathy associated with fever. This clinical scenario, including exposure to kittens, is suggestive of Bartonella henselae infection or “cat-scratch disease.” The adolescent in the vignette has lymphadenopathy, fever, headache, and abdominal pain as well as exposure to kittens, therefore, empiric treatment with azithromycin would be appropriate. Empiric treatment with azithromycin can be considered in situations compatible with this clinical syndrome (eg, acute lymphadenopathy and exposure to kittens). Penicillin, amoxicillin, and first-generation cephalosporins are not recommended for treatment of B henselae.
PREP Pearls
Cat-scratch disease is a common cause of lymphadenopathy in children, caused by the gram-negative bacillus Bartonella henselae.
The majority of Bartonella henselae infections are asymptomatic. The severity of disease varies based on the age and immune status of the host, ranging from regional lymphadenopathy to systemic disseminated infection.
Uncomplicated cases of lymphadenopathy due to Bartonella henselae do not require treatment. However, a 5-day course of azithromycin results in reduction of lymph node size and hastens resolution of symptoms.
A 14-year-old adolescent girl is seen for evaluation of a 9-kg weight gain over the past year. Her mother is concerned about a possible hormonal etiology. The adolescent reports fatigue and depression. She often stays up late working on her computer and will snack during this time. She does not sleep well and snores loudly. She reports occasional headaches, but no vision changes, nausea, or vomiting. She has no muscle weakness. Menarche occurred at age 11 years, and her menses are regular. Her mother recently underwent bariatric surgery.
On physical examination, the adolescent’s blood pressure is 132/86 mm Hg and heart rate is 86 beats/min. Her weight is 85 kg (98th percentile), height is 163 cm (65th percentile) and body mass index is 32 kg/m2 (98th percentile). She has hyperpigmentation around her neck and in her skin creases, and thin striae on her abdomen. The remainder of her physical examination findings are normal.
Of the following, the BEST next step in this adolescent’s evaluation is to
A. gather a detailed dietary history B. measure a serum morning cortisol level C. obtain magnetic resonance imaging of the abdomen D. obtain thyroid function tests
detailed diet history
A 5-day-old male neonate born at 36 weeks’ gestation is brought to the emergency department for evaluation of a 1-day history of poor feeding, decreased activity, and episodes of excessive crying. He is exclusively breastfed. The infant was discharged home 2 days after birth with a bilirubin level of 8 mg/dL (136.83 µmol/L). The mother’s blood group is O positive; she does not know the neonate’s blood type. On physical examination, the neonate has a high-pitched cry, marked jaundice, increased extensor tone in the upper and lower extremities, intermittent arching, and an exaggerated Moro reflex. There is no clonus. Laboratory results include a total bilirubin level of 32 mg/dL (547.33 µmol/L), a blood type of A positive, and a positive direct antiglobulin (Coombs) test result. The neonate is admitted, and phototherapy is started.
Of the following, the MOST important next step in the management of this neonate is
A. continued phototherapy B. double volume exchange transfusion C. intravenous fluid therapy D. partial volume exchange transfusion
The neonate in the vignette is a late preterm infant with severe hyperbilirubinemia. His physical examination findings are consistent with phases 1 to 2 of acute bilirubin neurotoxicity. To prevent further neurologic deterioration and decrease the risk of the long-term neurodevelopmental sequelae of kernicterus, a double volume exchange transfusion should be performed as soon as possible. For this neonate, the total bilirubin level for which exchange transfusion is indicated, is 22.2 mg/dL (379.7 µmol/L); this neonate’s level is well above that (Item C133A).
Almost all preterm neonates and approximately 60% of term neonates will develop physiologic jaundice. This jaundice occurs due to catabolism of red blood cells (RBCs) (the half-life of RBCs in neonates is 90 days compared with 120 days in adults), increased RBC volume, immature hepatic conjugation, and delayed establishment of feedings, leading to reduced excretion of bilirubin and increased enterohepatic circulation of bilirubin. Item C133B lists risk factors for severe hyperbilirubinemia in neonates; the neonate in the vignette’s risk factors include gestational age (late preterm), exclusive breastfeeding, and isoimmune hemolytic disease.
Phototherapy should continue before and after exchange transfusion but will not rapidly decrease the bilirubin level, which is critical to minimizing the neurotoxic effects of severe hyperbilirubinemia. Intravenous fluid is helpful to address dehydration, if present, but it is not a treatment for hyperbilirubinemia. Partial volume exchange transfusion is performed in neonates with chronic anemia for whom heart failure is a concern and in neonates with symptomatic polycythemia with a central hematocrit greater than 65% to 70% (0.65-0.70); it is not an appropriate treatment for neonatal hyperbilirubinemia.
PREP Pearls
A high level of bilirubin or an insufficient amount of serum protein to bind the bilirubin leads to circulating free bilirubin, which can easily penetrate the blood–brain barrier.
To prevent further neurologic deterioration and decrease the risk of the long-term neurodevelopmental sequelae of kernicterus, a double volume exchange transfusion should be performed as soon as possible when exchange transfusion is indicated.
Some risk factors for significant neonatal hyperbilirubinemia include predischarge total serum or transcutaneous bilirubin level close to phototherapy threshold, lower gestational age, exclusive breastfeeding, jaundice observed in the first 24 hours, inherited red blood cell disorders, phototherapy before discharge, hemolysis from any cause, macrosomic infant, and Down syndrome.
A new patient is seen for a health supervision visit. The child is healthy with a normal birth history. He is crawling and pulling himself up to stand using the couch. Occasionally, he will stand up unassisted in the middle of the floor. He uses his thumb and index finger to pick up a piece of breakfast cereal. He is able to find his pacifier when his mother hides it under a blanket. He says ”mama” and “uh-oh” appropriately. He will stop and look at his parents when they say “no.”
Of the following, this child’s development is MOST consistent with that of a
A. 9-month-old B. 12-month-old C. 15-month-old D. 18-month-old
The child in the vignette is at a developmental level that is most consistent with that of a 12-month-old.
The fine motor development of a 12-month-old child should include the ability to pick things up between thumb and pointer finger, like small bits of food. The gross motor development of a 12-month-old child should include cruising and taking first steps. Often a child this age will stand independently, demonstrating age-appropriate tone and balance, but may not yet be willing to attempt the actual motion of walking. At 15 to 18 months of age we would expect a child to be walking well independently and beginning to run and climb.
A 7-month-old boy is admitted to the hospital for dehydration, weight loss, vomiting, and diarrhea. The diarrhea began approximately 4 weeks ago. He has 8 to 10 nonbloody, watery stools daily and frequent diaper rashes, which have been difficult to clear despite frequent application of zinc oxide diaper cream. His diet consists of cow milk formula, pureed fruits and vegetables, and oat and wheat infant cereal. He recently began to have intermittent, nonbilious, nonbloody emesis and decreased oral intake (both solids and formula). On physical examination, the infant is pale and fussy but consolable. His weight is at the third percentile, length is at the 50th percentile, and weight for length is fifth percentile for age. He has tachycardia, a capillary refill time of 2 seconds, a soft and nondistended abdomen without hepatosplenomegaly, and perianal erythema. The remainder of his physical examination findings are normal.
The infant is given intravenous fluids and made nil per os. Stool infectious studies, including viral and bacterial testing, are negative. His diarrhea significantly improves after 24 hours of bowel rest.
Of the following, the BEST next step in this infant’s management is
A. fiber supplementation B. loperamide C. serum vasoactive intestinal peptide concentration D. upper endoscopy
The infant in the vignette has chronic diarrhea (duration >14 days) associated with failure to thrive and therefore requires further evaluation. For a child with this clinical presentation, without evidence of secretory diarrhea, upper endoscopy is the best management option. With further evaluation, he was diagnosed with allergic enteropathy.
Chronic diarrhea may occur because of the following mechanisms:
Osmotic diarrhea: unabsorbed intestinal solutes drive luminal retention of water (via osmotic gradient)
Examples: lactose intolerance, exocrine pancreatic insufficiency
Diarrhea improves when unabsorbed nutrients are removed from the diet (or in nil per os [NPO] status)
Secretory diarrhea: excessive intestinal electrolyte and/or fluid secretion
Examples: neuroendocrine tumors (eg, VIPoma, neuroblastoma), congenital chloride diarrhea
Diarrhea persists despite NPO status
Low stool osmolar gap:
Stool Total Osmolarity – 2 × [Stool Na+ + Stool K+]
Inflammatory diarrhea: may include both secretory and osmotic mechanisms because intestinal fluid secretion may be excessive and the inflamed bowel may not completely absorb intestinal solutes
Examples: infectious diarrhea (excessive intestinal fluid secretion), inflammatory bowel disease, celiac disease, and allergic enteropathy
Diarrhea may improve with NPO status
Diarrhea caused by dysmotility: occurs because of decreased intestinal transit time
Examples: chronic nonspecific diarrhea, irritable bowel syndrome
PREP Pearls
Chronic diarrhea may be classified based on the mechanism: osmotic, secretory, inflammatory, and dysmotility associated.
Infants with chronic diarrhea and failure to thrive require additional evaluation, including blood and stool studies; endoscopic evaluation may be useful to determine the cause.
An 8-year-old girl is seen for a health supervision visit. She was recently diagnosed with a left optic pathway glioma that is being monitored by her health care team. She is doing well in the third grade in a general education classroom. The girl’s physical examination findings are significant only for multiple café au lait macules (at least 6 greater than 5 mm). Sexual maturity rating is 1 for breast and pubic hair development.
Of the following, this girl’s MOST likely diagnosis is
A. Li-Fraumeni syndrome B. neurofibromatosis type 1 C. neurofibromatosis type 2 D. tuberous sclerosis complex
The girl in the vignette has neurofibromatosis type 1 (NF1). She meets the clinical criteria for NF1 with her optic pathway glioma and at least 6 café au lait macules larger than 5 mm. Neurofibromatosis type 1 is a tumor predisposition syndrome inherited in an autosomal dominant manner with complete penetrance. An example of a NF1 pedigree is shown in Item C136A. Clinical criteria for NF1, recently revised, are below: